contentsfile.etoosindia.com/sites/default/files/study...topic page no. theory 01 - 06 exercise - 1...

44
Topic Page No. Theory 01 - 06 Exercise - 1 07 - 24 Exercise - 2 25 - 33 Exercise - 3 34 - 38 Exercise - 4 39 - 41 Answer Key 42 - 43 Contents Chemical Kinetics Syllabus Chemical Kinetics : Rates of chemical reactions; Order of reactions; Rate constant; First order reactions; Temperature dependence of rate constant (Arrhenius equation). Name : ____________________________ Contact No. __________________ ETOOSINDIA.COM India's No.1 Online Coaching for JEE Main & Advanced 3rd Floor, H.No.50 Rajeev Gandhi Nagar, Kota, Rajasthan 324005 HelpDesk : Tel. 092142 33303

Upload: others

Post on 03-Sep-2020

10 views

Category:

Documents


0 download

TRANSCRIPT

Page 1: Contentsfile.etoosindia.com/sites/default/files/study...Topic Page No. Theory 01 - 06 Exercise - 1 07 - 24 Exercise - 2 25 - 33 Exercise - 3 34 - 38 Exercise - 4 39 - 41 Answer Key

Topic Page No.

Theory 01 - 06

Exercise - 1 07 - 24

Exercise - 2 25 - 33

Exercise - 3 34 - 38

Exercise - 4 39 - 41

Answer Key 42 - 43

Contents

Chemical Kinetics

SyllabusChemical Kinetics :

Rates of chemical reactions; Order of reactions; Rate constant; First order reactions; Temperature

dependence of rate constant (Arrhenius equation).

Name : ____________________________ Contact No. __________________

ETOOSINDIA.COMIndia's No.1 Online Coaching for JEE Main & Advanced

3rd Floor, H.No.50 Rajeev Gandhi Nagar, Kota, Rajasthan 324005HelpDesk : Tel. 092142 33303

Page 2: Contentsfile.etoosindia.com/sites/default/files/study...Topic Page No. Theory 01 - 06 Exercise - 1 07 - 24 Exercise - 2 25 - 33 Exercise - 3 34 - 38 Exercise - 4 39 - 41 Answer Key

CHEMICAL KINETICS # 1

CHEMICAL KINETICS

Rate/Velocity of chemical reaction :

The rate of change of concentration with time of different chemical species taking part in a chemical reaction isknown as rate of reaction of that species.

Rate = tc

=

sec.lit/mol = mol lit�1 time�1 = mol dm�3 time�1

Rate is always defined in such a manner so that it is always a positive quantity.

Types of Rates of chemical reaction :

For a reaction R P

Average rate = takentimeTotalionconcentratinchangeTotal

= tc

= �

t]R[

=

t]P[

Instantaneous rate : rate of reaction at a particular instant.

Rinstantaneous

= 0tlim

tc

= dtdc

= � dt

]R[d =

dt]P[d

Relation between reaction rates of different species involved in a reaction :

For the reaction : N2 + 3H

2 2NH

3

Rate of reaction = dt

]N[d 2 = dt3]H[d1 2 =

dt

]NH[d

21 3

Order of reaction :Let there be a reaction m

1A + m

2B products.

Now, if on the basis of experiment, we find thatR [A]P [B]q Where p may or may not be equal to m

1 and similarly q may or may not be equal to m

2.

p is order of reaction with respect to reactant A and q is order of reaction with respect to reactant B and(p + q) is overall order of the reaction.

Integrated rate laws :Zero order reactions :For a zero order reactionGeneral rate law is, Rate = k [conc.]º = constant

If C0 is the initial concentration of a reactant and C

t is the

concentration at time �t� then

Rate = k = 't'

CC t0 or kt = C0 � C

tor C

t = C

0 � kt

Unit of K is same as that of Rate = mol lit�1 sec�1.

First Order Reactions :

k = t

303.2 log

t

0CC

Half life time (t1/2)

t1/2

= k

2log303.2 =

kln2

= k

0.693

ETOOSINDIA.COMIndia's No.1 Online Coaching for JEE Main & Advanced

3rd Floor, H.No.50 Rajeev Gandhi Nagar, Kota, Rajasthan 324005HelpDesk : Tel. 092142 33303

Page 3: Contentsfile.etoosindia.com/sites/default/files/study...Topic Page No. Theory 01 - 06 Exercise - 1 07 - 24 Exercise - 2 25 - 33 Exercise - 3 34 - 38 Exercise - 4 39 - 41 Answer Key

CHEMICAL KINETICS # 2

Second order reaction :

tC1�

0C1

= kt

Pseudo first order reaction :A second order (or of higher order) reactions can be converted into a first order reaction if the other reactant istaken in large excess. Such first order reactions are known as pseudo first order reactions. For A + B Products [Rate = K [A]1 [B]1

k = )ba(t303.2

log )xb(a)xa(b

Table : Characteristics of Zero, First, Second and nth Order Reactions of the Type A Products

Zero Order First-Order Second-Order nth order

Differential Rate lawtÄ

]A[Ä = k[A]°

t]A[

= k[A]

t]A[

= k[A]2

n]A[ktÄ]A[Ä

(Integrated Rate law) [A]t = [A]

0 � kt In [A]

t = �kt + In [A]

0 t]A[1

= kt + 0]A[

1

kt)1n(

)A(

1

)A(

11n

01n

t

Linear graph [A]t v/s t In [A] v/s t ]A[

1 v/s t 1n

t )A(

1 v/s t

Half-life t1/2

= k2

]A[ 0 t1/2

= k693.0

t1/2

= 0]A[k

11n

02/1

)A(

1t

(depends on [A]0) (independent of [A]

0) (depends on [A]

0)

Methods to determine order of a reaction :By comparison of different initial rates of a reaction by varying the concentrationof one of the reactants while others are kept constant

r = k [A]a [B]b [C]c if [B] = constant[C] = constant

then for two different initial concentrations of A we have

10r = k [A0]1

a20r = k [A

0]2

a

a

20

10

0

0

]A[

]A[

r

r

2

1

Method of half lives :

The half lives of each order is unique so by comparing half lives we can determine order

for nth order reaction t1/2

1n0 ]R[

1

1n0

1n'0

'2/1

2/1

)R(

)R(

t

t

ETOOSINDIA.COMIndia's No.1 Online Coaching for JEE Main & Advanced

3rd Floor, H.No.50 Rajeev Gandhi Nagar, Kota, Rajasthan 324005HelpDesk : Tel. 092142 33303

Page 4: Contentsfile.etoosindia.com/sites/default/files/study...Topic Page No. Theory 01 - 06 Exercise - 1 07 - 24 Exercise - 2 25 - 33 Exercise - 3 34 - 38 Exercise - 4 39 - 41 Answer Key

CHEMICAL KINETICS # 3

Methods to monitor the progress of the reaction :Pressure measurement :Progress of gaseous reaction can be monitored by measuring total pressure at a fixed volume & temperature.

Let there is a 1st order reactionA(g) nB(g)

k =t

2.303 log

t0

0PnP1)(nP

Final total pressure after infinite time = Pf = nP

0

Formula is not applicable when n = 1, the value of n can be fractional also.Do not remember the formula but derive it for each question.

By titration method :By measuring the volume of titrating agent we can monitor amount of reactant remaining or amount of productformed at any time. It is the titre value . Here the milliequivalent or millimoles are calculated using valencefactors.

V0 = vol. of titrant used at t = 0 [this is exclusively for HCl.]

Vt = vol. of titrant used at �t�

V = vol. of titrant used at t =

k = t

303.2 log

t

0VVVV

Optical rotation measurement :It is used for optically active sample. It is applicable if there is at least one optically active species involvedin chemical reaction.

where are r0, r

t, r

are angle of optical rotation at time t = 0, t = t and t =

K = 0

t

Y Y2.303log

t r r

Effect of temperature on rate of reaction :In early days the effect of temperature on reaction rate was expressed in terms of temperature coefficientwhich was defined as the ratio of rate of reaction at two different temperature differing by 10ºC (usually these

temperatures were taken as 25ºC and 35ºC)

T.C. = t

tK

10K 2 to 3 ( for most of the reactions)

Arrhenius theory of reaction rate :

Products

ReactantsHR

HP

Ea1 Ea2

Threshold enthalpy or energy

H = H � H = Ea � Eap R 1 2

Enthalpy (H)

Progress of reaction (or reaction coordinate)

H = Summation of enthalpies of reactantsH = Summation of enthalpies of reactantsH = Enthalpy change during the reaction

Ea = Energy of activation of the forward reactionEa = Energy of activation of the backward reaction

R

P

1

2

ETOOSINDIA.COMIndia's No.1 Online Coaching for JEE Main & Advanced

3rd Floor, H.No.50 Rajeev Gandhi Nagar, Kota, Rajasthan 324005HelpDesk : Tel. 092142 33303

Page 5: Contentsfile.etoosindia.com/sites/default/files/study...Topic Page No. Theory 01 - 06 Exercise - 1 07 - 24 Exercise - 2 25 - 33 Exercise - 3 34 - 38 Exercise - 4 39 - 41 Answer Key

CHEMICAL KINETICS # 4

T1

T2Fraction of molecule

Ea

T2 > T

1

RT/Eae represents fraction of molecules K.E. having energy greater Ea

rate RT/Eae

dependence of rate on temperature is due to dependence of k on temperature.

RT/E

RT/E

a

a

Aekek

[Arrhenius equation]

General characteristics of catalyst :P.E. Ea

E'a

HR

HP Products

Reaction Coordinate

A catalyst drives the reaction through a low energy path andhence E

a is less. That is, the function of the catalyst is to

lower down the activation energy.E

a = Energy of activation in absence of catalyst.

E�a = Energy of activation in presence of catalyst.

Ea � E�

a = lowering of activation energy by catalyst.

Molecularity and Order :

The number of molecules that react in an elementary step is the molecularity of the elementary reaction.Molecularity is defined only for the elementary reactions and not for complex reactions. No elementary reactionsinvolving more than three molecules are known, because of very low probability of near-simultaneous collision ofmore than three molecules.The rate law for the elementary reaction

aA + bB products rate = k[A]a[B]b, where a + b = 1, 2 or 3.

For an elementary reaction, the orders in the rate law equal the coefficients of the reactants.

Mechanism of a reaction :Reactions can be divided intoElementary / simple / single stepComplex / multi-step

ELEMENTARY REACTION :

These reaction take place in single step without formation of any intermediate

T.S.

Ep

Er

P.E.

Reaction coordinates

For elementary reaction we can define molecularity of the reaction which is equal to no of molecules whichmake transition state or activated complex because of collisions in proper orientation and with sufficient energy

ETOOSINDIA.COMIndia's No.1 Online Coaching for JEE Main & Advanced

3rd Floor, H.No.50 Rajeev Gandhi Nagar, Kota, Rajasthan 324005HelpDesk : Tel. 092142 33303

Page 6: Contentsfile.etoosindia.com/sites/default/files/study...Topic Page No. Theory 01 - 06 Exercise - 1 07 - 24 Exercise - 2 25 - 33 Exercise - 3 34 - 38 Exercise - 4 39 - 41 Answer Key

CHEMICAL KINETICS # 5

COMPLEX REACTION :Reaction which proceed in more than two steps. or having some mechanism. ( sequence of elementary reactionin which any complex reaction procceds)

T.S.

intermediateE

Reaction coordinates

For complex reaction each step of mechanism will be having its own molecularity but molecularity of netcomplex reaction will not be defined.

CALCULATION OF RATE LAW/ ORDER

MECHANISM IN WHICH R.D.S. GIVENIf R.D.S. involves only reactant, product or catalyst on reactant siderate law of R.D.S. = rate law of reaction

RDS is having intermediate on reactant sideTo calculate order, we have to specify [ intermediate] in expression of rate law in terms of conc. of [R], [P] orcatalyst with the help of same equilibrium step given in mechanism.

MECHANISMS IN WHICH RDS NOT SPECIFIED :

STEADY STATE APPROXIMATION :

At steady state dt

]ermediate[intd = 0

COMPLICATIONS IN 1st ORDER REACTIONPARALLEL 1st ORDER REACTION OR COMPETING FIRST-ORDER REACTIONS

effT1

= 1T

1+

2T1

(remember)

]C[]B[

= 2

1

kk

(remember)

21

2a1aa kk

kEkEE 21

REVERSIBLE 1ST ORDER REACTION ( both forward and backward )

A B

t = 0 a 0t = t a �x xt = t

eq.a � x

eq.x

eq.

ETOOSINDIA.COMIndia's No.1 Online Coaching for JEE Main & Advanced

3rd Floor, H.No.50 Rajeev Gandhi Nagar, Kota, Rajasthan 324005HelpDesk : Tel. 092142 33303

Page 7: Contentsfile.etoosindia.com/sites/default/files/study...Topic Page No. Theory 01 - 06 Exercise - 1 07 - 24 Exercise - 2 25 - 33 Exercise - 3 34 - 38 Exercise - 4 39 - 41 Answer Key

CHEMICAL KINETICS # 6

x = bf

f

kkak t)kk( bfe1

(remember)

kf + k

b =

t1

n

xx

x

.eq

.eq(remember)

SEQUENTIAL 1ST ORDER REACTION OR CONSECUTIVE FIRST-ORDER REACTIONS

A 1kB 2k

C all first order equation

t = 0 a 0 0t a � x y z

y = 12

1

kkak

{ tk1e � tk2e } (remember)

tmax.

= 21 kk1

n2

1

kk

(remember)

CASE- k1 >> k

2

A 1kB 2k

C

CASE : k2 >> k

1

a

t

[C]

[A][B]

Conc

ETOOSINDIA.COMIndia's No.1 Online Coaching for JEE Main & Advanced

3rd Floor, H.No.50 Rajeev Gandhi Nagar, Kota, Rajasthan 324005HelpDesk : Tel. 092142 33303

Page 8: Contentsfile.etoosindia.com/sites/default/files/study...Topic Page No. Theory 01 - 06 Exercise - 1 07 - 24 Exercise - 2 25 - 33 Exercise - 3 34 - 38 Exercise - 4 39 - 41 Answer Key

CHEMICAL KINETICS # 7

PART - I : OBJECTIVE QUESTIONS

*Marked Questions are having more than one correct option.

Section (A) : Rate of Reaction

A-1. For a hypothetical reaction A L, the rate expression is -

rate = � dt

dC A

(A) Negative sign represent that rate is negative(B) Negative sign indicates to the decrease in the concentrations of reactant(C) Negative sign indicates the attractive forces between reactants(D) None of the above is correct

A-2. xA + yB zC. If dt

]A[d =

dt]B[d

= 1.5dt

]C[d = then x,y and z are :

(A) 1,1, 1 (B) 3, 2, 3 (C) 3, 3, 2 (D) 2, 2, 3

A-3. In the following reaction : xA yB

log

dt]A[d

= log

dt]B[d

+ 0.3

where �ve sign indicates rate of disappearance of the reactant. Thus, x : y is :

(A) 1 : 2 (B) 2 : 1 (C) 3 : 1 (D) 3 : 10

A-4. In the formation of sulphur trioxide by the contact process,2SO2 (g) + O2 (g) 2SO3 (g)

The rate of reaction is expressed as �d O

dt

( )2 = 2.5 x 104 mol L-1 sec-1

The rate of disappearance of (SO2) will be -(A) 5.0 x 10�4 mol L�1 s�1 (B) 2.25 x 10�4 mol L�1 s�1

(C) 3.75 x 10�4 mol L�1 s�1 (D) 50.0 x 10�4 mol L�1 s�1

A-5. Rate of formation of SO3 in the following reaction 2SO2 + O2 2SO3 is 100 g min�1. Hence rate of disappearanceof O2 is :(A) 50 g min-1 (B) 40 g min�1 (C) 200 g min�1 (D) 20 g min�1

A-6. A reaction follows the given concentration (M)�time graph. The rate for this reaction at 20 seconds will be:

0.5

0.4

0.3

0.2

0.1

0 20 40 60 80 100

Time/second

(A) 4 × 10�3 M s�1 (B) 8 × 10�2 M s�1 (C) 2 × 10�2 M s�1 (D) 7 × 10�3 M s�1

A-7. For the reaction : N2 + 3H2 2NH3. If the rate of disappearance of hydrogen is 1.8 × 103. What is therate of formation of ammonia. (mole per litre per sec.)(A) 1.8 × 103 (B) 1.2 × 103 (C) 2.7 × 103 (D) 0.9 × 103

ETOOSINDIA.COMIndia's No.1 Online Coaching for JEE Main & Advanced

3rd Floor, H.No.50 Rajeev Gandhi Nagar, Kota, Rajasthan 324005HelpDesk : Tel. 092142 33303

Page 9: Contentsfile.etoosindia.com/sites/default/files/study...Topic Page No. Theory 01 - 06 Exercise - 1 07 - 24 Exercise - 2 25 - 33 Exercise - 3 34 - 38 Exercise - 4 39 - 41 Answer Key

CHEMICAL KINETICS # 8

A-8. The rate of a reaction is expressed in different ways as follows ;+ 1/2(d[C]/dt) = � 1/3 (d[D]/dt) = + 1/4 (d[A]/dt) = � (d[B]/dt) The reaction is :

(A) 4 A + B 2C + 3D (B) B + 3D 4A + 2C

(C) 4A + 2B 2C + 3D (D) B + (1/2) D 4A + 3

A-9. ln the reaction; A + 2B 3C + D, which of the following expression does not describe changes in theconcentration of various species as a function of time :(A) {d [C] / dt} = � {3d [A] / dt} (B) {3d [D] / dt} = {d [C] / dt}(C) {3d [B] / dt} = � {2d [C] / dt} (D) {2d [B] / dt} = {d [A] / dt}

A-10. For a chemical reaction 2X + Y Z, the rate of appearance of Z is 0.05 mol L�1 per min. The rate ofdisappearance of X will be -(A) 0.05 mol L�1 per hour (B) 0.05 mol L�1 per min(C) 0.1 mol L�1 per min (D) 0.25 mol L�1 per min

Section (B) : Rate Law

B-1. What is the order of a reaction whoserate = KCA

3/2 CB�1/2 ?

(A) 2 (B)1 (C)� 1/2 (D) 3/2

B-2. The rate of certain hypothetical reaction A + B + C products, is given, by r = � dAdt

= K [A]½ [B]1/3

[C]1/4. The order of reaction is given by :(A) 1 (B) 1/2 (C) 2 (D) 13/12

B-3. The rate constant of nth order has units :(A) Litre1�n mol1-n sec�1 (B) Mol1�n litre1-n sec

(C) Mol n1 2 litre n2 sec�1 (D) Mole1�n litren-1 sec�1

B-4. The rate constant of reaction changes when :(A) Volume is changed (B) Concentrations of the reactants are changed(C) Temperature is changed (D) Pressure is changed

B-5. For which of the following, the units of rate constant and rate of the reaction are same -(A) First order reaction (B) Second order reaction(C) Third order reaction (D) Zero order reaction

B-6. For a chemical reaction, 2A + 2B C + D, the order of reaction is one with respect to A and one with

respect to B. The initial rate of the reaction is 4 × 10�2 mol L�1 s�1. When 50% of the reactants are convertedinto products, the rate of the reaction would become-(A) 2 × 10�2 mol L�1s�1 (B) 1 × 10�2 mol L�1s�1

(C) 4 × 10�2 mol L�1 s�1 (D) 2 × 10�1 mol L�1 s�1

B-7. For a gaseous reaction the rate equation is v = k[A][B]. If the volume of the gaseous system is suddenlyreduced to 1/3 of initial volume. The rate would become �(A) 1/9 times (B) 9 times (C) 1/6 times (D) 6 times

B-8. aA + bB Product, dx/dt = k [A]a [B]b . If concentration of A is doubled, rate is four times. If concentration

of B is made four times, rate is doubled. What is relation between rate of disappearance of A and that of B ?(A) � {d [A] / dt} = � {d [B] / dt} (B) � {d [A] / dt} = � {4 d [B] / dt}

(C) � {4 d [A] / dt} = � {d [B]/ dt} (D) None of these

ETOOSINDIA.COMIndia's No.1 Online Coaching for JEE Main & Advanced

3rd Floor, H.No.50 Rajeev Gandhi Nagar, Kota, Rajasthan 324005HelpDesk : Tel. 092142 33303

Page 10: Contentsfile.etoosindia.com/sites/default/files/study...Topic Page No. Theory 01 - 06 Exercise - 1 07 - 24 Exercise - 2 25 - 33 Exercise - 3 34 - 38 Exercise - 4 39 - 41 Answer Key

CHEMICAL KINETICS # 9

B-9. For the reaction, 2NO(g) + 2H2(g) N2(g) + 2H2O(g) the rate expression can be written in the followingways :{d [N2] / dt} = k1 [NO][H2] ; {d[H2O] / dt} = k[NO][H2] ; {� d[NO] / dt} = k1 [NO] [H2] ; {�d[H2] / dt} = k1 [NO][H2]The relationship between k, k1 , k1 and k1. is :(A) k = k1 = k1 = k1 (B) k = 2k1 = k1 = k1(C) k = 2k1 = k1 = k1 (D) k = k1 = k1 = 2 k1

B-10. For rate constant is numerically the same for three reactions of first, second and third order respectively.Which of the following is correct :(A) if [A] = 1 then r1 = r2 = r3 (B) if [A] < 1 then r1 > r2 > r3(C) if [A] > 1 then r3 > r2 > r1 (D) All

B-11. In acidic medium the rate of reaction between (BrO3)¯ and Br¯ ions is given by the expression.

� [d (BrO3�) /dt] = K [BrO3

� ] [Br¯] [H+]2 It means :(A) Rate constant of overall reaction is 4 sec�1

(B) Rate of reaction is independent of the concentration of acid(C) The changes in pH of the solution will not affect the rate(D) Doubling the concentration of H+ ions will increase the reaction rate by 4 times.

B-12. For the irreversible process, A + B products, the rate is first�order w.r.t. A and second�order w.r.t. B. If 1.0

mol each of A and B introduced into a 1.0 L vessel, and the initial rate was 1.0 × 10�2 mol L�1 s�1 , rate when halfreactants have been turned into products is :(A) 1.25 × 10�3 mol L�1 s�1 (B) 1.0 × 10�2 mol L�1 s�1

(C) 2.50 × 10�3 mol L�1 s�1 (D) 2.0 × 10�2 mol L�1 s�1

Section (C) : The integrated rate laws

C-1. A first order reaction follows the expressions

(A) Cttk1e = C0 (B) Ct = C0

tk1e (C) lnt

0

CC

= �k1t (D) ln0

t

C

C = k1t

C-2. Which of following statement is false(A) A fast reaction has a larger rate constant and short half-life(B) For a first order reaction, successive half lives are equal(C) For a first order reaction, the half-life is independent of concentration(D) The half life of a reaction is half the time required for the reaction to go to completion

C-3. Half life period of a zero order reaction is �(A) Independent of concentration(B) Directly proportional to initial concentration(C) Inversely proportional to concentration(D) Directly proportional to the square of the concentration

C-4. Wrong data for the first order reaction is �(A) t0.5 =100s, t0.75 = 200s (B) t0.75 = 32 min t0.5 = 16min(C) Both the above (D) t0.5 = 100s, t0.75 = 150s

C-5. What is the half life of a radioactive substance if 75% of any given amount of the substance disintegratesin 60 minutes �(A) 2 Hours (B) 30 Minutes (C) 45 Minutes (D) 20 Minutes

C-6. For an elementary reaction X (g) Y (g) + Z (g) the half life periods is 10 min. In what period of timewould the concentration of X be reduced to 10% of original concentration -(A) 20 Min. (B) 33 Min (C) 15 Min (D) 25 Min

C-7. 99% of a first order reaction was completed in 32 min. When will 99.9% of the reaction complete �(A) 50 Min (B) 46 Min (C) 49 Min (D) 48 Min

ETOOSINDIA.COMIndia's No.1 Online Coaching for JEE Main & Advanced

3rd Floor, H.No.50 Rajeev Gandhi Nagar, Kota, Rajasthan 324005HelpDesk : Tel. 092142 33303

Page 11: Contentsfile.etoosindia.com/sites/default/files/study...Topic Page No. Theory 01 - 06 Exercise - 1 07 - 24 Exercise - 2 25 - 33 Exercise - 3 34 - 38 Exercise - 4 39 - 41 Answer Key

CHEMICAL KINETICS # 10

C-8. In the presence of an acid, the initial concentration of cane sugar was reduced from 0.20 to 0.10 molarin 5 hours and from 0.2 to 0.05 molar in 10 hours. The reaction is of �(A) Zero order (B) First order (C) Second order (D) Third order

C-9. For a second order reaction, if the conc. of a reactant decreases from 0.08M to 0.04M in ten minutes,what would be the time taken for the conc. to decreases to 0.01M �(A) 20 minutes (B) 30 minutes (C) 50 minutes (D) 70 minutes

C-10. The rate constant for a second order reaction 8.0 × 10�4 litre mol�1min�1. How long will it take a 0.5Msolution to be reduced to 0.25M in reactant �(A) 8.665 × 102 min (B) 8.0 × 10�4 min (C) 2.50 × 103 min (D) 4.0 × 10�4 min

C-11. A radioactive isotope decomposes according to the first order with half life period of 15 hrs. 80% of thesample will decompose in -(A) 15 × 0.8 hr. (B) 15 × (log 8) hr.

(C) 15 × (log5 / log2) hr. (or 34.83) hr. (D) 15 × 10/ 8 hr.

C-12. Radioactive decay follows -(A) Zero order kinetics (B) Second order kinetics(C) First order kinetics (D) Pseudo first order kinetics

C-13. The t0.5 for the first order reaction,

PCI5(g) PCI3(g) + Cl2(g)is 20 min. The time in which the conc. of PCI5 reduces to 25% of the initial conc. is close to-(A) 22 min (B) 40 min (C) 90 min (D) 50 min

C-14. The rate constant of reaction 2 A + B C is 2.57 × 10�5 It mole�1 sec�1 after 10 sec. 2.65 × 10�5 It.mole�1 sec�1 after 20 sec. and 2.55 × 10�5 It. mole�1 sec�1 after 30 sec. The order of the reaction is:(A) 0 (B) 1 (C) 2 (D) 3

C-15. For a first order reaction, the plot of log C against �t� (logC vs 't') gives a straight tine with slope equal to :

(A) (k / 2.303) (B) (� k / 2.303) (C) (ln k / 2.303) (D) � k.

C-16. In a certain reaction, 10% of the reactant decomposes in one hour, 20 % in two hours, 30% in three hours andso on the dimensions of the rate constant is :(A) hour�1 (B) mole litre�1 sec�1 (C) litre mole�1 sec�1 (D) mole sec�1

C-17. In presence of HCl, sucrose gets hydrolysed into glucose and fructose. The concentration of sucrose was foundto reduce from 0.4 M to 0.2 M in 1 hour and to 0.1 M in total of 2 hours. The order of the reaction is :(A) zero (B) one (C) two (D) None of these

C-18. In a first order reaction the reacting substance has half-life period of ten minutes. What fraction of the substancewill be left after an hour the reaction has occurred ?(A) 1/6 of initial concentration (B) 1/64 of initial concentration(C) 1/12 of initial concentration (D) 1/32 of initial concentration

C-19. In the following first order reactions (A) 1K Product, (B) 2K

Product, the ratio k1 /k2 if 90% of (A) has

been reacted in time 't' while 99% of(B) has been reacted in time 2t is :(A) 1 (B) 2 (C) 1/2 (D) None of these

C-20. Two substances A (t1/2 = 5 min) and B (t1/2 = 15 min) are taken in such a way that initially [A] = 4[B]. The timeafter which both the concentration will be equal is : (Assume that reaction is first order)(A) 5 min (B) 15 min(C) 20 min (D) concentration can never be equal

ETOOSINDIA.COMIndia's No.1 Online Coaching for JEE Main & Advanced

3rd Floor, H.No.50 Rajeev Gandhi Nagar, Kota, Rajasthan 324005HelpDesk : Tel. 092142 33303

Page 12: Contentsfile.etoosindia.com/sites/default/files/study...Topic Page No. Theory 01 - 06 Exercise - 1 07 - 24 Exercise - 2 25 - 33 Exercise - 3 34 - 38 Exercise - 4 39 - 41 Answer Key

CHEMICAL KINETICS # 11

C-21. If a I-order reaction is completed to the extent of 60% and 20% in time intervals, t1 and t2, what is the ratio,t1 : t2 ?(A) 6.32 (B) 5.58 (C) 4.11 (D) 8.33

C-22. For a reaction A Products, the concentration of reactant are C0 , aC0 , a2C0, a

3C0 ........... after time interval0, t, 2t ........where 'a' is constant. Then :(A) reaction is of 1st order and K = (1/1) ln a (B) reaction is of 2nd order and K = (1/tC0)(1�a)/a

(C) reaction is of 1st order and K = t1

ln

a1

(D) reaction is of zero order and K =

a1

lnt1

C-23. The rate constant for the reaction A B is 2 × 10�4 It. mol�1 min�1. The concentration of A at which rate of thereaction is (1/12) × 10�5 M sec�1 is :

(A) 0.25 M (B) (1/20) 3/5 M (C) 0.5 M (D) None of these

C-24. Graph between concentration of the product and time of the reaction A B is of the type Hence graph

between � d[A]/dt and time will be of the type :

(A)

(�d[A]/dt)

Time

(B) (C) (D)

C-25. A graph plotted between log t50% vs. log concentration is a straight line. What conclusion can you draw from thisgraph.

(A) n = 1 ; t1/2 a (B) n = 2, t1/2 1/a(C) n = 1 ; t1/2 = (0.693 / k) (D) None of these

C-26. What will be the order of reaction and rate constant for a chemical change having log t50% vs log concentrationof (A) curves as :(A) 0, 1/2 (B) 1, 1 (C) 2, 2 (D) 3, 1

C-27. A drop of solution (volume 0.05 mL) contains 3.0 × 10�6 moles of H+. If the rate constant of disappearance of H+

is 1.0 × 107 mole litre�1 sec�1. How long would it take for H+ in drop to disappear :(A) 6 × 10�8 sec (B) 6 × 10�7 sec (C) 6 × 10�9 sec (D) 6 × 10�10 sec

Section (D) : Methods to determine The rate law

D-1. When concentration of reactant in reactionA B is increased by 8 times, the rate increase only 2 times. The order of the reaction would be -(A) 2 (B) 1/3 (C) 4 (D) 1/2

D-2. The data for the reaction A + B C isExp. [A]0 [B]0 initial rate

1 0.012 0.035 0.102 0.024 0.035 0.803 0.012 0.070 0.104 0.024 0.070 0.80

(A) r = k [B]3 (B) r = k [A]3 (C) r = k [A] [B]4 (D) r = k [A]2 [B]2 .

ETOOSINDIA.COMIndia's No.1 Online Coaching for JEE Main & Advanced

3rd Floor, H.No.50 Rajeev Gandhi Nagar, Kota, Rajasthan 324005HelpDesk : Tel. 092142 33303

Page 13: Contentsfile.etoosindia.com/sites/default/files/study...Topic Page No. Theory 01 - 06 Exercise - 1 07 - 24 Exercise - 2 25 - 33 Exercise - 3 34 - 38 Exercise - 4 39 - 41 Answer Key

CHEMICAL KINETICS # 12

D-3. A + B Product, dtdx

=k [A]a [B]b

If

dtdx

= k, then order is :

(A) 4 (B) 2 (C) 1 (D) 0

D-4. One litre of 2M acetic acid and one litre of 3M ethyl alcohol were mixed. The esterification takes placeaccording to the reaction:CH3COOH + C2H5OH CH3COOC2H5 + H2OIf each solution is diluted by one litre water the rate would become �(A) 4 times (B) 2 times (C) 0.5 times (D) 0.25 times

D-5. In the reaction : A + 2B + C D + 2ETher rate of reaction remains unchanged if the conc. of B is doubled and that of A and C is kept constant.What is the order with respect to B.(A) 0 (B) 1/2 (C) 1 (D) 3

D-6. The following data pertain to reaction between A and B :S.No. [A] [B] Rate

mol.l�1 mol.l�1 mol.l�1sec�1

I 1 × 10�2 2 × 10�2 2 × 10�4

II 2 × 10�2 2 × 10�2 4 × 10�4

III 2 × 10�2 4 × 10�2 8 × 10�4

Which of the following inference(s) can be drawn from the above data �(a) Rate constant of the reaction 10�4

(b) Rate law of the reaction is k[A][B](c) Rate of reaction increases four times on doubling the concentration of both the reactant,Select the correct answer �codes :�(A) a,b and c (B) a and b (C) b and c (D) c alone

D-7. Using the data given below the order and rate constant for the reaction : CH3CHO(g) CH4(g) + CO(g) wouldbe �

Experiment no. Initial conc. Initial rate[mol/] [mol.lit�1sec1]

I 0.10 0.020II 0.20 0.080III 0.30 0.180IV 0.40 0.320

Answer is �(A) 2,[k = 2.0 /mol sec] (B) 0,[k = 2.0 mol/ sec](C) 2,[k = 1.5 /mol sec] (D) 1,[k = 1.5 sec�1]

D-8. For the reaction 2A + 3B products, A is in excess and on changing the concentration of B from 0.1 M to0.4 M, rate becomes doubled, Thus, rate law is :

(A) dtdx

= k[A]2 [B]2 (B) dtdx

= k[A] [B] (C) dtdx

= k[A]0 [B]2 (D)dtdx

= k[B]1/2

D-9. For the reaction A Products, � dt

]A[d = k and at different time interval, [A] values are :

Time 0 5 min 10 min 15 min[A] 20 mol 18 mol 16 mol 14 mol

At 20 minute, rate will be :(A) 12 mol /min (B) 10 mol/min (C) 8 mol/min (D) 0.4 mol/min

ETOOSINDIA.COMIndia's No.1 Online Coaching for JEE Main & Advanced

3rd Floor, H.No.50 Rajeev Gandhi Nagar, Kota, Rajasthan 324005HelpDesk : Tel. 092142 33303

Page 14: Contentsfile.etoosindia.com/sites/default/files/study...Topic Page No. Theory 01 - 06 Exercise - 1 07 - 24 Exercise - 2 25 - 33 Exercise - 3 34 - 38 Exercise - 4 39 - 41 Answer Key

CHEMICAL KINETICS # 13

D-10. A Product and

dtdx

= k[A]2 . If log

dtdx is plotted against log [A], then graph is of the type :

(A) (B) (C) (D)

D-11. If a is the initial concentration of reaction, then the half life period of a reaction of nth order is directlypropotional to -(A) an (B) an�1 (C) a1�n (D) an+1

D-12. The half-life period for a reaction at initial concentrations of 0.5 and 1.0 moles litre-1 are 200 sec and 100sec respectively. The order of the reaction is -(A) 0 (B) 1 (C) 2 (D) 3

D-13. Consider a reaction A B + C. If the initial concentraton of A was reduced from4 M to 2 M in 1 hour and from 2 M to 1 M in 0.5 hour, the order of the reaction is-(A) One (B) Zero (C) Two (D) Three

Section (E) : Methods to monitor the progress of reaction

E-1. Consider the reaction 2A(g) 3B(g) + C(g). Starting with pure A initially, the total pressure doubled in 3 hrs.The order of the reaction might possibly be(A) zero (B) first (C) second (D) unpredictable from this data

E-2. Formation of NO2F from NO2 and F2 as per the reaction 2NO2(g) + F2(g) 2NO2F(g) is a second orderreaction, first order with respect to NO2 and first order with respect to F2. If NO2 and F2 are present in a closedvessel in ratio 2 :1 maintained at a constant temperature with an initial total pressure of 3 atm, what will be thetotal pressure in the vessel after the reaction is complete?

(A) 1atm (B) 2 atm (C) 2.5 atm (D) 3 atm

E-3. In a gaseous state reaction, A2 (g) B(g) + (1/2)C (g), The increase in pressure from 100 mm to 120 mmis notices in 5 minutes. The rate of disapearance of A2 in mm min�1 is :(A) 4 (B) 8 (C) 16 (D) 2

E-4. The decomposition of a gaseous substance (A) to yield gaseous products (B), (C) follows First order kinetics.If initially only (A) is present and 10 minutes after the start of the reaction the pressure of (A) is 200 mm Hg andthat of over all mixture is 300 mm Hg, then the rate constant for 2A B + 3 C is :(A) (1/1200) ln 1.25 sec�1 (B) (2.303 /10) log 1.5 min�1

(C) (1/10) ln 1.25 sec�1 (D) None of these

E-5. In the reaction NH4NO2 (aq.) N2 (g) + 2 H2O (l) the volume of N2 after 20 min and after a long time is 40 ml and70 ml respectively. The value of rate constant is :(A) (1/20) In (7/4) min�1 (B) (2.303 /1200) log (7/3) sec�1

(C) (1/20) log (7/3) min�1 (D) (2.303 / 20) log (11/7) min�1

E-6. �N2Cl Cu/ �Cl + N2 Half-life is independent of concentration of reactant. After 10 minutes

volume of N2 gas is 10 L and after complete reaction it is 50 L. Hence rate constant is:(A) (2.303 /10) log 5 min�1 (B) (2.303 /10) log 1.25 min�1

(C) (2.303 /10) log 2 min�1 (D) (2.303 /10) log 4 min�1

ETOOSINDIA.COMIndia's No.1 Online Coaching for JEE Main & Advanced

3rd Floor, H.No.50 Rajeev Gandhi Nagar, Kota, Rajasthan 324005HelpDesk : Tel. 092142 33303

Page 15: Contentsfile.etoosindia.com/sites/default/files/study...Topic Page No. Theory 01 - 06 Exercise - 1 07 - 24 Exercise - 2 25 - 33 Exercise - 3 34 - 38 Exercise - 4 39 - 41 Answer Key

CHEMICAL KINETICS # 14

Section(F) : Effect of Temperature

F-1. The activation energies of the forward and backward reactions in the case of a chemical reaction are 30.5and 45.4 KJ/mol respectively. The reaction is �(A) Exothermic (B) Endothermic(C) Neither exothermic nor endothermic (D) Independent of temperature

F-2. Chemical reaction occurs as a result of collisions between reacting molecules. Therefore, the reaction rateis given by -(A) Total number of collisions occurring in a unit volume per second(B) Fraction of molecules which possess energy less than the threshold energy(C) Total number of effective collisions(D) None of the above

F-3. The chemical reactions in which reactants require high amount of activation energy are generally -(A) Slow (B) Fast (C) Instantaneous (D) Spontaneous

F-4. For a reaction for whi ch the activation energies of forward and reverse reactions are equal -(A) H = 0 (B) S = 0 (C) The order is zero (D) There is no catalyst

F-5. The activation energy of reaction is equal to-(A) Threshold energy for the reaction(B) Threshold energy + Energy of the reactants(C) Threshold energy � Energy of the reactants

(D) Threshold energy + Energy of the products

F-6. Which of the following explains the increase of the reaction rate by catalyst -(A) Catalyst decreases the rate of backward reaction so that the rate of forward reaction increases(B) Catalyst provides extra energy to reacting molecules so that they may reduce effective collisions(C) Catalyst provides an alternative path of lower activation energy to the reactants(D) Catalyst increases the number of collisions between the reacting molecules.

F-7. The activation energy of the reactionA + B C + D + 38 k.cal is 20 k.cal, What would be the activation energy of the reaction,C + D A + B(A) 20 k.cal (B) �20 k.cal (C) 18 k.cal (D) 58 k.cal

F-8. Rate of which reactions increases with temperature :(A) of any (B) of exothermic reactions(C) of endothermic (D) of none.

F-9. For a zero order reaction. Which of the following statement is false :(A) the rate is independent of the temperature of the reaction.(B) the rate is independent of the concentration of the reactants.(C) the half life depends upon the concentration of the reactants.(D) the rate constant has the unit mole It�1 sec�1.

F-10. A large increase in the rate of a reaction for a rise in temperature is due to(A) increase in the number of collisions (B) the increase in the number of activated molecules(C) The shortening of mean free path (D) the lowering of activation energy

F-11. The first order rate constant k is related to temp. as log k = 15.0 � (106 /T) Which of the following pair of value iscorrect ?(A) A = 1015 and E = 1.9 × 104 KJ (B) A = 10�15 and E = 40 KJ(C) A = 1015 and E = 40 KJ (D) A = 10�15 and E = 1.9 × 104 KJ.

F-12. The decomposition of N2O into N2 & O2 in presence of gaseous argon follow second order kinetics with

k = (5.0 × 1011 L mol1 s1) TK41570

e

(K stands for Kelvin units). The energy of activation of the reaction is :

(A) 5.0 x 1011 J (B) 41570 J (C) 5000 J (D) 345612.98 J

ETOOSINDIA.COMIndia's No.1 Online Coaching for JEE Main & Advanced

3rd Floor, H.No.50 Rajeev Gandhi Nagar, Kota, Rajasthan 324005HelpDesk : Tel. 092142 33303

Page 16: Contentsfile.etoosindia.com/sites/default/files/study...Topic Page No. Theory 01 - 06 Exercise - 1 07 - 24 Exercise - 2 25 - 33 Exercise - 3 34 - 38 Exercise - 4 39 - 41 Answer Key

CHEMICAL KINETICS # 15

F-13. How much faster would a reaction proceed at 25°C than at 0°C if the activation energy is 65 kJ?

(A) 2 times (B) 5 times (C) 11 times (D) 16 times

F-14. The rate constant, the activation energy and the frequency factor of a chemical reaction at 25°C are

3.0 × 10�4 s�1, 104.4 KJ mol�1 and 6.0 × 1014 s�1 respectively. The value of the rate constant as T is :(A) 2.0 × 1018 s�1 (B) 6.0 × 1014 s�1 (C) infinite (D) 3.6 × 1030 s�1

F-15. For a given reaction, energy of activation for forward reaction (Eaf) is 80kJ.mol1. H = 40kJ.mol1 for thereaction. A catalyst lowers Eaf to 20 kJ.mol1. The ratio of energy of activation for reverse reaction before andafter addition of catalyst is :(A) 1.0 (B) 0.5 (C) 1.2 (D)2.0

Section (G) : Mechanism of reactionsG-1. For the reaction H2 (g) + Br2 (g) 2HBr (g) the experiment data suggested that r = k[H2][Br2]

1/2

The molecularity and order of the reaction are respectively :(A) 2, 3/ 2 (B) 3/2 , 3/2 (C) Not defined, 3/2 (D) 1,1/2

G-2. For the reaction NO2 + CO CO2 + NO the experimental rate expression is � dtdc

= k[NO2]2 the number of

molecules of CO involves in the slowest step will be -(A) 1 (B) 2 (C) 3 (D) depends on mechanism

G-3. The reaction of hydrogen, and iodine monochloride is represented by the equation :H2(g) + 2Cl(g) 2HCl(g) + 2(g)

This reaction is first�order in H2(g) and also first�order in Cl(g). Which of these proposed mechanismcan be consistent with the given information about this reaction ?

Mechanism : H2(g) + 2Cl(g) 2HCl(g) + 2(g)

Mechanism : H2(g) + Cl(g) Slow HCl(g) + H(g)

HI(g) + Cl(g) fast HCl(g) + I2(g)

(A) only (B) only (C) both and (D) neither nor

G-4. The reaction, X + 2Y + Z N occurs by the following mechanism(i) X + Y M very rapid equilibrium(ii) M + Z P slow(iii) O + Y N very fastWhat is the rate law for this reaction

(A) Rate = k[Z] (B) Rate = k[X] [Y]2 [Z] (C) Rate = [N] (D) Rate = k[X] [Y] [Z]

G-5. In the Lindemann theory of unimolecular reactions, it is shown that the apparent rate constant for such a

reaction is kapp = C1

Ck1

where C is the concentration of the reactant k1 and are constants. Calculate the

value of C for which kapp has 90% of its limiting value at C tending to infinitely large values, given = 9 x 105.(A) 106 mole/litre (B) 104 mol/litre (C) 105 mole/litre (D) 5 x 105 mol/litre

G-6. Trimolecular reactions are uncommon because(A) the probability of three molecules colliding at an instant is very low.(B) the probability of three molecules colliding at an instant is high.(C) the probability of three molecules colliding at an instant is zero.(D) the probability of many molecules colliding at an instant is high.

G-7. Select the correct statements :(A) the molecularity of an elementary reaction indicates how many reactant molecules take part in the step.(B) the rate law of an elementary reaction can be predicted by simply seeing the stoichiometry of reaction.(C) the slowest elementary step in sequence of the reactions governs the overall rate of formation of product.(D) a rate law is often derived from a proposed mechanism by imposing the steady state approximation or

assuming that there is a pre-equilibrium.

ETOOSINDIA.COMIndia's No.1 Online Coaching for JEE Main & Advanced

3rd Floor, H.No.50 Rajeev Gandhi Nagar, Kota, Rajasthan 324005HelpDesk : Tel. 092142 33303

Page 17: Contentsfile.etoosindia.com/sites/default/files/study...Topic Page No. Theory 01 - 06 Exercise - 1 07 - 24 Exercise - 2 25 - 33 Exercise - 3 34 - 38 Exercise - 4 39 - 41 Answer Key

CHEMICAL KINETICS # 16

Section (H) : Complications in first order reactions

H-1. For an elementary reaction, net rate is

dtdx

= k[A]2 � k'[C] [B]2 then, select the correct statement :

(A) �dt

]A[d 2=

dt]B[d 2

=dt

]C[d is the relation among (B) 2A 2B + C is the required reaction

(C) both are correct (D) none is correct

H-2. Consider the elementary reaction sequence shown in figure. Which of the following

equations are correct ?

(A) dt

]A[d= k1[A] + k4[D] (B)

dt]C[d

= k2[B] k3[C]

(C) dt

]D[d = k4[D] + k3[D] (D) Nothing can be said about order of reactions in this problem

H-3. For an elementary reaction of reversible nature, net rate is :

dtdx

= k1 [A]2 [B]1 � k2 [C], hence, given reaction is :

(A) 2A + B1

C (B) 2A + B C (C) 2A C + B�1 (D) None of these

H-4. At a given temperature, k1 = k2 for the reaction

A + B C + D

If

dtdx

= k1 [A] [B] � k2[C] [D] in which set of the concentration reaction ceases?

[A] [B] [C] [D] [A] [B] [C] [D]

(A) 0.1 M 0.2 M 0.3 M 0.4 M (B) 0.4 M 0.25 M 0.2 M 0.5 M

(C) 0.2 M 0.2 M 0.3 M 0.2 M (D) 0.2 M 0.2 M 0.4 M 0.2 M

H-5. The substance undergoes first order decomposition. The decomposition follows two parallel first order reactions

as :

K1 = 1.26 × 10�4 sec�1 and K2 = 3.8 × 10�5 sec�1

The percentage distribution of B and C

(A) 80% B and 20% C (B) 76.83% B and 23.17%C

(C) 90% B and 10% C (D) 60% B and 40% C

H-6. The rate constant for two parallel reactions were found reactions were found to be 1.0 × 10�2dm3 mol-1 s�1 and

3.0 × 10�2 dm3 mol�1 s�1. If the corresponding energies of activation of the parallel reactions are 60.0 kJ mol�1

and 70.0 kJ mol�1 respectively, what is the apparent overall energy of activation ?

(A) 130.0 kJ mol�1 (B) 67.5 kJ mol�1 (C) 100.0 kJ mol�1 (D) 65.0 kJ mol�1

ETOOSINDIA.COMIndia's No.1 Online Coaching for JEE Main & Advanced

3rd Floor, H.No.50 Rajeev Gandhi Nagar, Kota, Rajasthan 324005HelpDesk : Tel. 092142 33303

Page 18: Contentsfile.etoosindia.com/sites/default/files/study...Topic Page No. Theory 01 - 06 Exercise - 1 07 - 24 Exercise - 2 25 - 33 Exercise - 3 34 - 38 Exercise - 4 39 - 41 Answer Key

CHEMICAL KINETICS # 17

PART - II : MISCELLANEOUS QUESTIONS

COMPREHENSION

Comprehension # 1

A(g) 2B(g) + C(g)

Initially at t = 0 gas A was present along with some amount of gas (C). At t = 0 mole fraction of gas C is

1/3. After some time t = t1, total pressure is half of the final total pressure at t = t

x (a very long time). Assume this

decomposition is a first order, at a constant temperature. It is also given at t = tx, final total pressure is 35 bar.

1. At t = t1 pressure of gas B is :

(A) 2.5 bar (B) 1.25 bar (C) 5.0 bar (D) data is insufficient

2. Rate constant (k) = (log 64 � log 49) s�1. Value of t1 in seconds is :

(A) 2.15 s (B) 1.5 s (C) 2.3 s (D) 1.15 s

3. Ratio of rate constant at t = 0 to t = t1 to t = t

x is :

(A) 2 : 3 : 4 (B) 1 : 1 : 1 (C) 1 : 3 : 5 (D) 1 : 3 : 5

Comprehension # 2

4. For the (Set-1) :

(A) if T1 > T

2, k

1 > k

2 always (B) if T

1 > T

2, k

1 > k

2 (for exothermic reaction)

(C) if T1 > T

2, k

1 < k

2 (for endothermic reaction) (D) Ea

1 Ea

2

5. For the (Set-1) :

(A) Ea1 > Ea

2if T

1 > T

2(B) Ea

1 < Ea

2if T

1 > T

2

(C) Ea1 = Ea

2(D) Ea

1 = 0.5 Ea

2

6. Comparing set-I and II :

(A) k4 > k

3 & k

2 > k

1 , if T

2 > T

1 (endothermic) (B) k

4 < k

3 & k

2 > k

1 , if T

2 < T

1 (endothermic)

(C) k4 > k

3 & k

2 > k

1 , if T

2 < T

1 (exothermic) (D) k

4 < k

3 & k

2 < k

1, if T

2 > T

1 (exothermic)

ETOOSINDIA.COMIndia's No.1 Online Coaching for JEE Main & Advanced

3rd Floor, H.No.50 Rajeev Gandhi Nagar, Kota, Rajasthan 324005HelpDesk : Tel. 092142 33303

Page 19: Contentsfile.etoosindia.com/sites/default/files/study...Topic Page No. Theory 01 - 06 Exercise - 1 07 - 24 Exercise - 2 25 - 33 Exercise - 3 34 - 38 Exercise - 4 39 - 41 Answer Key

CHEMICAL KINETICS # 18

Comprehension # 3A reaction is said to be first order if it's rate is proportional to the concentration of reactant. Let us consider areaction

A(g) B(g) + C(g)At t = 0 a 0 0At time t a � x x x

The rate of reaction is given by the expression dt

dx = k(a � x) and integrated rate equation for a given reaction is

represented as k = t

1 ln

xa

awhere a = initial concentration and (a � x) = concentration of A after time t.

7. Thermal decomposition of compound X is a first order reaction. If 75% of X is decomposed in 100 min. How longwill it take for 90% of the compound to decompose?Given : log 2 = 0.30(A) 190 min (B) 176.66 min (C) 166.66 min (D) 156.66 min

8. Consider a reaction A(g) 3B(g) + 2C(g) with rate constant 1.386 × 10�2 min�1. Starting with2 moles of A in 12.5 litre vessel initially, if reaction is allowed to takes place at constant pressure & at 298K thenfind the concentration of B after 100 min.(A) 0.04 M (B) 0.36 M (C) 0.09 M (D) None of these

Comprehension # 4Competing First-Order ReactionsFrequently a species can react in different ways to give a variety of products. For example, toluene can benitrated at the ortho, meta, or para positions, We shall consider the simplest case, that of two competingirreversible first-order reactions :

A 1k C and A 2k Dwhere the stoichiometric coefficients are taken as unity for simplicity. The rate law is

dt

]A[d= � k

1[A]

� k

2[A]

= � (k

1 + k

2) [A] [A] = [A]

0t)kk( 21e .

For C, we have

dt]C[d

= k1[A] = k

1[A]

0t)kk( 21e . Multiplication by dt and integration from time 0

(where [C]0 = 0) to an arbitary time t gives [C] = )e1(

kk

]A[k t)kk(

21

01 21

Similarly, integration of

dt]D[d

gives [D] = 21

02

kk

]A[k

(1 � t)kk( 21e )

The sum of the rate constants k1 + k

2 appears in the exponentials for both [C] and [D].

At any time we also have ]D[]C[

= 2

1

kk

9. A starting initially with only A Which of the following is correct at time t

(A) [A]0 = [A]

t +[B]

t + [C]

t(B) [A]

0 = [A]

t + 2 [B]

t + 3 [C]

t

(C) [A]0 = [A]

t +

2

]B[ t + 3

]C[ t (D) [A]0 =

32

[A]t +[B]

t + [C]

t

ETOOSINDIA.COMIndia's No.1 Online Coaching for JEE Main & Advanced

3rd Floor, H.No.50 Rajeev Gandhi Nagar, Kota, Rajasthan 324005HelpDesk : Tel. 092142 33303

Page 20: Contentsfile.etoosindia.com/sites/default/files/study...Topic Page No. Theory 01 - 06 Exercise - 1 07 - 24 Exercise - 2 25 - 33 Exercise - 3 34 - 38 Exercise - 4 39 - 41 Answer Key

CHEMICAL KINETICS # 19

10. X starting with only 'X', ratio tt

t

]Z[]Y[

]X[

(A) Independent of time (B) )1e(

1kt

(C) Depends upon initial concentration of X (D) [A]0 (ekt �1)

11. At high temperature acetic acid decomposes into CO2 & CH

4 and simultaneously into CH

2CO (ketene) and H

2O

(i) CH3COOH

11 s3k CH

4 + CO

2(ii) CH

3COOH

12 s4k CH

2CO + H

2O

What is the fraction of acetic acid reacting as per reaction (i) ?

(A)43

(B) 73

(C) 74

(D) none of these

12. For A starting with pure A ratio of rate of production of B to C is

(A) Independent of time (B) Independent of temperature(C) Depends upon initial concentration of A (D) Independent of mechanism of reaction

Comprehension # 5

For the given sequential reaction

A 1k B 2k C

the concentration of A, B & C at any time 't' is given by

[A]t = tk0

1e]A[ ; [B]t = )kk(

]A[k

12

01

tktk 21 ee

[C]t = [A0] � ( [A]t + [B]t )

13. The time at which concentration of B is maximum is

(A) 12

1

kk

k

(B)

2

1

12 k

kln

kk

1

(C)

2

1

21 k

kln

kk

1

(D)

12

2

kk

k

14. Select the correct option if k1 = 1000 s�1 and k2 = 20 s�1.

(A)

[C]t

[B]t[A]t

conc

.

time

(B)

[A]t[B]t

[C]t

Con

c.

time

(C)

[A]t[B]t

[C]t

Con

c.

time

(D)

[A]t

[B]t

[C]t

Con

c.

time

ETOOSINDIA.COMIndia's No.1 Online Coaching for JEE Main & Advanced

3rd Floor, H.No.50 Rajeev Gandhi Nagar, Kota, Rajasthan 324005HelpDesk : Tel. 092142 33303

Page 21: Contentsfile.etoosindia.com/sites/default/files/study...Topic Page No. Theory 01 - 06 Exercise - 1 07 - 24 Exercise - 2 25 - 33 Exercise - 3 34 - 38 Exercise - 4 39 - 41 Answer Key

CHEMICAL KINETICS # 20

MATCH THE COLUMN

15. The polarimeter readings in an experiment to measure the rate of inversion of cane suger (1st order reaction)were as follows

time (min) : 0 30

angle (degree) : 30 20 � 15

Then match the following. (Use : log 35 = 1.54, log 3 = 0.48, log 2 = 0.3)

Column - I Column - II

(A) The half life of the reaction (p) 120 min.

(B) The solution is optically inactive at (q) 7.5°

(C) The equimolar mixture of the products (r) 75.2 min.

(D) The angle at half time (s) laevorotatory

16. Match the following :Column - I Column - II

(A) A + B C + D (p) Unit of rate constant possess concentrationr = k

1 [A] [B] unit

(B) A + B C + D (q) Rate constant for the reaction of bothr = k

2 [A] [B]º the reactants are equal

(C) A + B C + D (r) Rate of consumption of at least one of ther = k

3 [A]º [B]º reactants is equal to rate of production of at

least one of the products

(D) 2A + B 2C + 3D (s) If both reactants are taken in stoichiometricr = k

3 [A]º [B]º ratio, half life for both reactants are equal

17. For the reaction of type A(g) 2B(g)

Column-I contains four entries and column-II contains four entries. Entry of column-I are to be matched

with ONLY ONE ENTRY of column-II

Column - I Column - II

(A)dt

]B[d vs

dt

]A[d for first order (p)

(B) [A] vs t for first order (q)

(C) [B] vs t for first order (r)

(D) [A] vs t for zero order (s)

ETOOSINDIA.COMIndia's No.1 Online Coaching for JEE Main & Advanced

3rd Floor, H.No.50 Rajeev Gandhi Nagar, Kota, Rajasthan 324005HelpDesk : Tel. 092142 33303

Page 22: Contentsfile.etoosindia.com/sites/default/files/study...Topic Page No. Theory 01 - 06 Exercise - 1 07 - 24 Exercise - 2 25 - 33 Exercise - 3 34 - 38 Exercise - 4 39 - 41 Answer Key

CHEMICAL KINETICS # 21

18. Match the following :

Column-I Column-II

(A) If the activation energy is 65 kJ then how much time (p) 2faster a reaction proceed at 25°C than at 0°C

(B) Rate constant of a first - order reaction is 0.0693 min�1. (q) ZeroIf we start with 20 mol L�1, it is reduced to 2.5 mol L�1

in how many minutes

(C) Half - lives of first - order and zero order reactions (r) 11are same. Ratio of rates at the start of reaction ishow many times of 0.693 Assume initial concentration tobe same for the both.

(D) The half-life periods are given , (s) 30[A]

0(M) 0.0677 0.136 0.272

t1/2

(sec) 240 480 960order of the reaction is

ASSERTION / REASONDirections :

Each question has 5 choices (A), (B), (C), (D) and (E) out of which ONLY ONE is correct.(A) Statement-1 is True, Statement-2 is True; Statement-2 is a correct explanation for Statement-1.(B) Statement-1 is True, Statement-2 is True; Statement-2 is NOT a correct explanation for Statement-1.(C) Statement-1 is True, Statement-2 is False.(D) Statement-1 is False, Statement-2 is True.(E) Statement-1 and Statement-2 both are False.

19. Statement-1 : A fractional order reaction must be a complex reaction.Statement-2 : Fractional order of RDS equals to overall order of a complex reaction.

20. Statement-1 : The time of completion of reactions of type A product (order <1) may be determined.

Statement-2 : Reactions with order 1 are either too slow or too fast and hence the time of completion can

not be determined.

21. Statement-1 : Temperature coefficient of an one step reaction may be negative.Statement-2 : The rate of reaction having negative order with respect to a reactant decreases with the

increase in concentration of the reactant.

22. Statement-1 : The overall rate of a reversible reaction may decrease with the increase in temperature.Statement-2 : When the activation energy of forward reaction is less than that of backward reaction, then the

increase in the rate of backward reaction is more than that of forward reaction on increasing the temperature.

23. Statement-1 : In a reversible endothermic reaction, Eact

of forward reaction is higher than that of backward reaction

Statement-2 : The threshold energy of forward reaction is more than that of backward reaction

24. Statement-1 : A catalyst provides an alternative path to the reaction in which conversion ofreactants into products takes place quickly

Statement-2 : The catalyst forms an activated complex of lower potential energy, with thereactants by which more number of molecules are able to cross the barrier per unit of time.

ETOOSINDIA.COMIndia's No.1 Online Coaching for JEE Main & Advanced

3rd Floor, H.No.50 Rajeev Gandhi Nagar, Kota, Rajasthan 324005HelpDesk : Tel. 092142 33303

Page 23: Contentsfile.etoosindia.com/sites/default/files/study...Topic Page No. Theory 01 - 06 Exercise - 1 07 - 24 Exercise - 2 25 - 33 Exercise - 3 34 - 38 Exercise - 4 39 - 41 Answer Key

CHEMICAL KINETICS # 22

25. Statement-1 : In the reaction, 2 + 2S

2O

32� S

4O

62� + 2� , the rate of disappearance of thiosulphate ions is

twice the rate of disappearance of 2.

Statement-2 : The rate of disappearance of 2 is twice the rate of disappearance of S

2O

32� ions.

26. Statement-1 : For A + 2B C (rate= K[A]1[B]0), the half life time of reaction is only defined when conc.

of A and B are in stoichiometric ratio

Statement-2 : For above given order half life of reaction is directly proportional to conc. of A and not to conc.

of B due to its zero order.

27. Statement-1 : Many reactions occurring on solid surface are zero order reactions.

Statement-2 : 22Au

2 O2/1)g(N)g(ON ; rate = k[N2O]0 = k = constant is a zero order reaction.

28. Statement-1 : Half life of a certain radioactive element is 100 days. After 200 days, fraction left undecaye

will be 25%.

Statement-2 : n

0

t21

CC

, where symbols have standard meaning.

29. Statement-1 : Time taken for the completion of 75% of a st order reaction is double than its t½

.

Statement-2 : Time taken for completion of any fraction of a st order reaction is a constant value.

30. Statement-1 : If the activation energy of reaction is zero temperature will have no effect on the rate constant

Statement-2 : Lower the activation energy faster is the reaction

TRUE / FALSE

31. Order of a reaction can be written from the balanced chemical equation.

32. In a complex reaction the rate of overall reaction is governed by the slowest step.

33. If t1/2 for a first order reaction is 6.93 s, the value of rate constant for the reaction would be 10s�1.

34. Molecularity of a reaction is always whole number.

35. Order and molecularity of a single step reaction may or may not be same.

36. The activation energy of a catalysed reaction is more than the activation energy of the uncatalysed reaction.

37. For a zero order reaction t3/4 is related to t1/2 as t3/4 = 1.5 t1/2

38. The rate of an exothermic reaction increases with the rise in temperature.

39. Partial orders are never negative.

40. Order of reaction may be change with change in practical conditions

41. The rate of reaction is uniform in zero order reaction.

ETOOSINDIA.COMIndia's No.1 Online Coaching for JEE Main & Advanced

3rd Floor, H.No.50 Rajeev Gandhi Nagar, Kota, Rajasthan 324005HelpDesk : Tel. 092142 33303

Page 24: Contentsfile.etoosindia.com/sites/default/files/study...Topic Page No. Theory 01 - 06 Exercise - 1 07 - 24 Exercise - 2 25 - 33 Exercise - 3 34 - 38 Exercise - 4 39 - 41 Answer Key

CHEMICAL KINETICS # 23

42. The rate law of the elementary reaction; 3A B must be r = k [A]3

43. If the partial orders are equal to corresponding coefficients in the balanced reaction, the reaction must be

elementary.

44. Every species that appears in the rate law of reaction must be a reactant or product in that reaction.

45. Molecularity defined only for RDS.

46. Product can form only when the required orientation and energy conditions are met.

47. Activated complex is an intermediate product.

48. The pre-exponential factor A has the same units for all reactions.

49. All collisions between reactants yield the desired product.

50. If the partial orders differ from the coefficients in the balanced reaction, the reaction must be complex.

51. The rate of the reaction A B having the rate law

]B[]A[k

dt]A[d

when plotted against time will exhibit a

maximum at some time.

52. For a firstorder reaction, the time required to reduce successively the concentration of reactant by a constant

fraction is always same.

53. As a first order reaction proceeds at a constant temperature, the rate remains constant.

54. If concentration of catalyst appear in rate law (rate = k [catalyst]1 [reactant]1) then it may becomes pseudo first

order reaction during the reaction.

55. Larger the value of Ea , greater is the effect on the value of k for a given temperature change.

56. A catalyst in a chemical reaction decreases the activation energy of the forward reaction and increases the

activation energy of the reverse reaction

57. A catalyst in a chemical reaction increases the forward Ea and decreases the backward E

a

58. A catalyst in a chemical reaction decreases both forward and backward Ea

59. At lower temperature, increase in temperature cause more change in the value of k than at higher temperature.

60. Molecularity of a reaction may includes the number of product molecules taking part in the reaction.

ETOOSINDIA.COMIndia's No.1 Online Coaching for JEE Main & Advanced

3rd Floor, H.No.50 Rajeev Gandhi Nagar, Kota, Rajasthan 324005HelpDesk : Tel. 092142 33303

Page 25: Contentsfile.etoosindia.com/sites/default/files/study...Topic Page No. Theory 01 - 06 Exercise - 1 07 - 24 Exercise - 2 25 - 33 Exercise - 3 34 - 38 Exercise - 4 39 - 41 Answer Key

CHEMICAL KINETICS # 24

FILL IN THE BLANKS

61. For collision to be effective the energy possessed by the colliding molecules should be equal to or greater thanthe ____________.

62. In the reaction, H2 + I2 2HI, the rate of disappearance of H2 is _______ the rate of appearance of HI.

63. For an endothermic process, the minimum value of activation energy can be _______.

64. The rate of a reaction is ________ to the collision frequency.

65. The rate constant for the zero order reaction has the dimensions___________.

66. The reactions with molecularity more than three are _________.

67. A catalyst increases the rate of the reaction by__________ activation energy.

68. If activation energy of reaction is low, it proceeds at _______ rate.

69. In a multistep reaction, the ________ step is rate determining.

70. Rate constant of a reaction __________ with increase in temperature.

71. The ratio t7/8 / t1/2 for a first order reaction would be equal to _________.

72. For a zero order reaction, the rate of the reaction is equal to the ______ of the reaction.

73. The value of temperature coefficient is generally between _________.

74. For a certain reaction, xM yL, the rate of reaction increases by 4 times when the concentration of M is

doubled. The rate law is _________.

75. The rate equation r = k [A][B]1/2 suggests that order of overall reaction is _______.

76. A plot of [A] vs t for a certain reaction A B with r = k [A]0 will be a straight line with slope equal to ________.

77. [Eactivated complex � Ereactants] = ____________.

78. In the Arrhenius equation k = Aexp (�E/RT), A may be termed as the rate constant at __________.

79. For a ______ order reaction the half-life (t1/2) is independent of the initial conc. of the reactants.

80. For a first order reaction A P, a graph of log [A] vs t has a slope equal to __________.

81. A catalyst is chemically ____________ at the end of a reaction

82. The order of a reaction is the ________ of the _________ of all the concentration terms in the rate equation.

ETOOSINDIA.COMIndia's No.1 Online Coaching for JEE Main & Advanced

3rd Floor, H.No.50 Rajeev Gandhi Nagar, Kota, Rajasthan 324005HelpDesk : Tel. 092142 33303

Page 26: Contentsfile.etoosindia.com/sites/default/files/study...Topic Page No. Theory 01 - 06 Exercise - 1 07 - 24 Exercise - 2 25 - 33 Exercise - 3 34 - 38 Exercise - 4 39 - 41 Answer Key

CHEMICAL KINETICS # 25

PART - I : MIXED OBJECTIVE

Single Choice Type

1. Consider the reaction :A B + C

Initial concentration of A is 1 M. 20 minutes time is required for completion of 20 % reaction.

If dt

]B[d = k[A], then half life (t1/2 ) is

(A) 55.44 min. (B) 50 min (C) 62.13 min (D) None of these

2. If decomposition reaction A (g) B (g) follows first order kinetics then the graph of rate of formation (R) of Bagainst time t will be

(A) (B) (C) (D)

3. For the first order decomposition of SO2Cl2(g),SO2Cl2(g) SO2(g) + Cl2(g)

a graph of log (a0 � x) vs t is shown in figure. What is the rate constant (sec�1)?

-1�

-2�

-3�

2 4 6 8 10| | | | |

log

(a �

x)

0

Time (min)

(0,0)

(A) 0.2 (B) 4.6 × 10�1 (C) 7.7 × 10�3 (D) 1.15 × 10�2

4. The rate constant for the forward reaction A (g) + 2B(g) is 1.5 × 10�3 s�1 at 100 K. If 10�5 moles of A and 100moles of B are present in a 10 litre vessel at equilibrium then rate constant for the backward reaction at thistemperature is :(A) 1.50× 104 L mol�1 s�1 (B) 1.5 × 1011 L mol�1 s�1

(C) 1.5 × 1010 L mol�1 s�1 (D) 1.5 × 10�11L mol�1 s�1

5. Reaction A + B C + D follows following rate law : rate = 21

21

]B[]A[k

. Starting with initial conc. of 1 M of

A and B each, what is the time taken for concentration of A of become 0.25 M.Given : k = 2.303 × 10�3 sec�1.(A) 300 sec. (B) 600 sec. (C) 900 sec. (D) 1200 sec.

6. The reaction A (g) B(g) + 2C (g) is a first order reaction with rate constant 3465 × 10�6 s�1. Starting with0.1 mole of A in 2 litre vessel, find the concentration of A after 200 sec., when the reaction is allowed to takeplace at constant pressure and temperature.(A) 0.05 M (B) 0.025 M (C) 0.0125 M (D) None of these

ETOOSINDIA.COMIndia's No.1 Online Coaching for JEE Main & Advanced

3rd Floor, H.No.50 Rajeev Gandhi Nagar, Kota, Rajasthan 324005HelpDesk : Tel. 092142 33303

Page 27: Contentsfile.etoosindia.com/sites/default/files/study...Topic Page No. Theory 01 - 06 Exercise - 1 07 - 24 Exercise - 2 25 - 33 Exercise - 3 34 - 38 Exercise - 4 39 - 41 Answer Key

CHEMICAL KINETICS # 26

7. The variation of concentration of A with time in two experiments starting with two different initial concentration ofA is given in the following graph. The reaction is represented as A(aq) B(aq). What is the rate of reaction(M/min) when concentration of A in aqueous solution was 1.8 M?

0.6

0.8

1

1.2

1.5

5 10 15 20

Experiment-1Experiment-2

time(min.) C

once

ntra

tion(

M)

(A) 0.08 M min�1 (B) 0.036 M min�1 (C) 0.13 M min�1 (D) 1 M min�1

8. In respect of the equation k = A exp (� Ea / RT), which one of the following statements is correct?(A) R is Rydberg's constant (B) k is equilibrium constant(C) A is adsorption factor (D) Ea is the energy of activation

9. Rate of a reaction can be expressed by Arrhenius equation as :k = Ae�E/RT

In this equation, E represents(A) The fraction of molecules with energy greater than the activation energy of the reaction(B) The energy above which all the colliding molecules will react(C) The energy below which colliding molecules will not react(D) The total energy of the reacting molecules at a temperature, T

10. The rate constant, the activation energy and the Arrhenius parameter (A) of a chemical reaction at 25°C are

3.0 × 10�4 s�1, 104.4 kJ mol�1 and 6.0 × 1014s�1 respectively. The value of the rate constant at T is(A) 2.0 × 1018 s�1 (B) 6.0 × 1014 s�1 (C) infinity (D) 3.6 × 1030 s�1

11. A first order reaction is 50% completed in 20 minutes at 27°C and in 5 min at 47°C. The energy of activation of

the reaction is(A) 43.85 kJ/mol (B) 55.14 kJ/mol (C) 11.97 kJ/mol (D) 6.65 kJ/mol

12. For the first order reaction A � B + C, carried out at 27 ºC if 3.8 × 10�16 % of the reactant molecules existsin the activated state, the Ea (activation energy) of the reaction is [log 3.8 = 0.58](A) 12 kJ/mole (B) 831.4 kJ/mole (C) 100 kJ/mole (D) 88.57 kJ/mole

13. For a certain reaction of order n, the time for half change, t1/2, is given by t1/2 =2/1

0xCk

)22( where k is

constant and C0 is the initial concentration. What is n?(A) 1 (B) 2 (C) 0 (D) 0.5

14. Given that for a reaction of order n, the integrated form of the rate equation is k =

1n

01n C

1

C

1)1n(t

1 where C0

and C are the values of the reactant concentration at the start and after time t. What is the relationship betweent3/4 and t1/2 where t3/4 is the time required for C to become 1/4 C0?(A) t3/4 = t1/2 [2

n1 + 1] (B) t3/4 = t1/2 [2n1 1]

(C) t3/4 = t1/2 [2n+1 1] (D) t3/4 = t1/2 [2

n+1 + 1]

ETOOSINDIA.COMIndia's No.1 Online Coaching for JEE Main & Advanced

3rd Floor, H.No.50 Rajeev Gandhi Nagar, Kota, Rajasthan 324005HelpDesk : Tel. 092142 33303

Page 28: Contentsfile.etoosindia.com/sites/default/files/study...Topic Page No. Theory 01 - 06 Exercise - 1 07 - 24 Exercise - 2 25 - 33 Exercise - 3 34 - 38 Exercise - 4 39 - 41 Answer Key

CHEMICAL KINETICS # 27

15. In three different reactions, involving a single reactant in each case, a plot of rate of the reaction on the yaxis,versus concentration of the reactant on the xaxis, yields three different curves shown below.

(i) (ii) (iii)

What are the possible orders of the reactions (i), (ii), (iii).

(A) 1, 2, 3 (B) 2, 1, 1/2 (C) 0, 1, 2 (D) 0, 1, 1/2

16. The time elapsed of a certain between 33% and 67% completion of a first order reaction is 30 minutes. What isthe time needed for 25% completion?(A) 150.5 minutes (B) 12.5 minutes (C) 180.5 minutes (D) 165.5 minutes

17. A reaction 2A + B k C + D is first order with respect to A and 2nd order with respect to B. Initial conc.

(t = 0) of A is C0 while B is 2C0. If at t as 30 minutes the conc. of C is C0 /4 then rate expression at t = 30 minutes is :

(A) R = 7 C03 k/16 (B) R = 27 C0

3 k / 32 (C) R = 247 C03 k / 64 (D) R = 49 k C0

3 / 32

18. For a certain reaction, a plot of {[C0 � C] / [C]} against the time t, yields a straight line. C0 = initial conc. of thereactant and C is the reactant concentration after time t. What is the order of the reaction :(A) 3 (B) zero (C) 1 (D) 2

19. In a I order reaction A products, the concentration of the reactant decrease to 6.25% of its initial value in 80minutes. What is (i) the rate constant and (ii) the rate of the reaction, 100 minutes after the start, if the initialconcentration is 0.2 mole/litre?(A) 2.17 × 102 min1, 3.47 × 104 mol.litre1 min1

(B) 3.465 × 102 min1, 2.166 × 104 mol.litre1 min1

(C) 3.465 × 103 min1, 2.17 × 103 mol.litre1 min1

(D) 2.166 × 103 min1, 2.667 × 104 mol.litre1 min1

20. At 373 K, a gaseous reaction A 2B + C is found to be of first order. Starting with pure A, the total pressure atthe end of 10 min. was 176 mm and after a long time when A was completely dissociated, it was 270 mm. Thepressure of A at the end of 10 minutes was :(A) 94 mm (B) 47 mm (C) 43 mm (D) 90 mm

21. The reaction A(s) 2 B(g) + C(g) is first order. The pressure after 20 min. and after very long time are 150 mmHg and 225 mm Hg. The value of rate constant and pressure after 40 min. are :(A) 0.05 In 1.5 min�1, 200 mm (B) 0.5 ln 2 min�1 ,300 mm(C) 0.05 In 3 min�1, 300 mm (D) 0.05 In 3 min�1 , 200 mm

22. The reaction [Co (NH3)5Br]2 + H2O [Co(NH3)5(H2O)]3+ + Br is followed by measuring a property of thesolution known as the optical density of the solution which may be taken to be linearly related to the concentrationof the reactant. The values of optical density are 0.80, 0.35 and 0.20 at the end of 20 minutes, 40 minutes andinfinite time after the start of the reaction which is first order. Calculate the rate constant.

(A) 6.93 x 103 min1 (B) 3.51 x 102 min1 (C) 6.93 x 102 min1 (D) 3.51 x 103 min1

23. The inversion of cane sugar proceeds with half life of 50 minute at pH = 5 for any concentration of sugar. Howeverif pH = 6, the half life changes to 500 minute of any concentration of sugar . The law expression for sugarinversion can be written as :

(A) r = K [sugar]2 [H+]0 (B) r = K [sugar]1 [H+]0 (C) r = K [sugar]1 [H+]1 (D) r = K [sugar]0 [H+]0.

ETOOSINDIA.COMIndia's No.1 Online Coaching for JEE Main & Advanced

3rd Floor, H.No.50 Rajeev Gandhi Nagar, Kota, Rajasthan 324005HelpDesk : Tel. 092142 33303

Page 29: Contentsfile.etoosindia.com/sites/default/files/study...Topic Page No. Theory 01 - 06 Exercise - 1 07 - 24 Exercise - 2 25 - 33 Exercise - 3 34 - 38 Exercise - 4 39 - 41 Answer Key

CHEMICAL KINETICS # 28

24. The reaction A k Products, is zero order while the reaction B

k Products, is 1st order. For what initial

concentration of A the half lives of the two reactions are equal.(A) 2 M (B) ln 2 M (C) 2 log 2 M (D) 2 ln 2 M

25. For the reaction 3A Products the value of k = 1 × 10�3 / (mol�min) the value of � d[A] / dt in

mol/lt-sec when [A] = 2M is :(A) 6.67 × 10�3 (B) 1.2 × 10�2 (C) 2 × 10�4 (D) 4 × 10�3

26. Two I-order reactions have half-lives in the ratio 3 : 2. Calculate the ratio of time intervals t1 : t2. Where t1 is thetime period for 25% completion of the first reaction and t2, for 75% completion of the second reaction.(A) 0.311 : 1 (B) 0.420 : 1 (C) 0.273 : 1 (D) 0.199 : 1

27. The Arrhenius relationship of two different reactions is shown below. Which reaction isfaster at a lower temperature and which is more sensitive to changes of temperature ?(A) B faster, A more sensitive (B) B in both cases(C) A in both cases (D) A faster, B more sensitive

28. When a graph between log K and 1/T is drawn a straight line is obtained. The point at which line cuts y -axis andx -axis respectively correspond to the temp :(A) 0, Ea / 2.303 R log A (B) , Ea / (R ln A)(C) 0, log A (D) None of these

29. A B KA = 1015 e�2000/T ;C D KC = 1014 e�1000/T

Temperature T K at-which (KA = KC) is :(A) 1000K (B) 2000K (C) (2000 / 2.303) K (D) (1000 / 2.303) K.

30. The rate of a reaction gets doubled when the temperature changes from 7ºC to 17ºC. By what factor will it

change for the temperature change from 17ºC to 27ºC?

(A) 1.81 (B) 1.71 (C) 1.91 (D) 1.76

31. For the reaction A + 2B products(started with concentrations taken in stoichiometric proportion), the

experimentally determined rate law is � dt

]A[d = k ]A[ ]B[ . The half time of the reaction would be :

(A) k693.0

(B) k/1

693.0 (C) k2

693.0(D) not defined

32. For a reaction 2A + B product, rate law is �dt

]A[d = k[A]. At a time when t =

k1

, concentration of the reactant

is : (C0 = initial concentration)

(A)e

C0 (B) C0e (C) 20

e

C(D)

0C1

33. Concentration of the reactant in first-order is reduced to 2e

1after : (Natural life =

K1

)

(A) one natural life-time (B) two-natural life-time(C) three natural life-time (D) four natural life-time

34. If for a reaction in which A(g) converts to B(g) the reaction carried out at const.V & T results into the following graph.

(A) then the reaction must be A(g) 3B(g) and is a first order reaction.(B) then the reaction must be A(g) 3B(g) and is a second order reaction.(C) then the reaction must be A(g) 3B(g) and is a zero order reaction.(D) then the reaction must be A(g) 3B(g) and is a first order reaction.

ETOOSINDIA.COMIndia's No.1 Online Coaching for JEE Main & Advanced

3rd Floor, H.No.50 Rajeev Gandhi Nagar, Kota, Rajasthan 324005HelpDesk : Tel. 092142 33303

Page 30: Contentsfile.etoosindia.com/sites/default/files/study...Topic Page No. Theory 01 - 06 Exercise - 1 07 - 24 Exercise - 2 25 - 33 Exercise - 3 34 - 38 Exercise - 4 39 - 41 Answer Key

CHEMICAL KINETICS # 29

35. The instantaneous rate of disappearance of the MnO4� ion in the following reaction is 4.56 × 10�3 ms�1. Then the

rate of appearance of 2 is :2MnO4

� + 10 � + 16H+ 2Mn2+ + 52 + 8H2O2MnO4

� + 10 � + 16H+ 2Mn2+ + 52 + 8H2O(A) 1.14 × 10�3 Ms�1 (B) 5.7 × 10�3 Ms�1 (C) 4.56 × 10�4 Ms�1 (D) 1.14 × 10�2 Ms�1

36. Consider the following reactions at 300 K. A B (uncatalysed reaction). BA catalyst (catalyst reaction)

The activation energy is lowered by 8.314 KJ mol�1 for the catalysed reaction. How many times the rate of thiscatalysed reaction greater than that of uncatalysed reaction ? (Given e3.33 = 28)(A) 15 times (B) 38 times (C) 22 times (D) 28 times

37. The conversion of vinyl allyl ether to pent-4-enol follows first-order kinetics.The following plot is obtained for such a reaction. The rate constant for thereaction is(A) 4.6 × 10�2 s�1

(B) 1.2 × 10�2 s�1

(C) 2.3 × 10�2 s�1

(D) 8.4 × 10�2 s�1

38. A substance �A' decomposes in solution following the first order kinetics. Flask contains 1 L of 1 M solution ofA and flask contains 100 ml of 0.6 M solution. After 8 hr, the conc. of A in flask 1 becomes 0.25 M. What willbe time for cone. of A in flask to become 0.3 M ? :(A) 0.4 hr (B) 2.4 hr(C) 4.0 hr (D) unpredictable as rate constant is not given

39. A simple mechanism for enzyme-catalyzed reaction is given by the following set of equations

E + S ES (enzyme) (reactant) (intermediate-1)

ES EP (intermediate-1) (intermediate-2)

EP E + P (intermediate-2) (enzyme) (product)This is known as the Michaelis�Menten mechanism. The potential energy diagram is shown in the fig. Which

of the following sets of identifications is correct ? (Assume that the temperature and pressure are constant).(1) (2) (3) (4)

(A) E + P. EP ES E + S(B) ES Activated complex EP Activated complex(C) EP Activated complex ES Activated complex(D) E + S ES EP E + P

More than one choice Type

40. For the reaction A B, the rate law expression istd

]A[d = k [A]1/2. If initial concentration of [A] is [A]0, then

(A) The integerated rate expression is k = )AA(t

2 2/12/10

(B) The graph of A Vs t will be

(C) The half life period 2/1t = 2/10]A[2

K

(D) The time taken for 75% completion of reaction 4/3t = k

]A[ 0

ETOOSINDIA.COMIndia's No.1 Online Coaching for JEE Main & Advanced

3rd Floor, H.No.50 Rajeev Gandhi Nagar, Kota, Rajasthan 324005HelpDesk : Tel. 092142 33303

Page 31: Contentsfile.etoosindia.com/sites/default/files/study...Topic Page No. Theory 01 - 06 Exercise - 1 07 - 24 Exercise - 2 25 - 33 Exercise - 3 34 - 38 Exercise - 4 39 - 41 Answer Key

CHEMICAL KINETICS # 30

41. Select incorrect statement(s):(A) Unit of pre-exponential factor (A) for second order reaction is mol L�1 s�1.(B) A zero order reaction must be a complex reaction.(C) Molecularity is defined only for RDS in a complex reaction.(D) Decay constant () of radioactive substance is affected by temperature.

42. In a consecutive reaction system A 1E B 2E C when E1 is much greater than E2, the yield ofB increase with

(A) increase in temperature (B) decreases in temperature

(C) increase in initial concentration of A (D) decrease in initial concentration of A

43. Which of the following is/are correct statement?(A) Stoichiometry of a reaction tells about the order of the elementary reactions.(B) For a zero order reaction, rate and the rate constant are identical.(C) A zero order reaction is controlled by factors other than concentration of reactants.(D) A zero order reaction is always elementary reaction.

44. Which of the following statement is incorrect ?(A) The order of reaction is the sum of powers of all the concentration terms in the rate equation.(B) The order of reaction with respect to one reactant is the ratio of the change of logarithm of the rate of thereaction to the change in the logarithm of the concentration of the particular reactant, keeping the concentrationsof all other reactants constant.(C) Orders of reactions can not be fractional.(D) The order of a reaction can only be determined from the stoichiometric equation for the reaction.

45. Consider the decay of P to A and B by two parallel first order reactions as shown in Fig. Given

Reaction H Rate constant Energy of activation

P A HA kA EA

P B HB kB EB

Which of the following is(are) true?(A) a = EB (B) b = EA (C) HA = b d (D) HB = c a

PART - II : SUBJECTIVE QUESTIONS

1. The oxidation of iodide ion by peroxy disulphate ion is described by the equation :

3� (aq) + S2O82� (aq) 3

� (aq) + 2SO42� (aq)

(a) If �t

]OS[ 282

= 1.5 × 10�3 Ms�1 for a particular time interval, what is the value of �t

][

for the same time

interval ?(b) What is the average rate of formation of SO4

2� during that time interval ?

2. In the following reaction 2H2O2 (aq) 2H2O () + O2 (g) rate of formation of O2 is 36 g min�1 ,

(a) What is rate of formation of H2O.(b) What is rate of disappearance of H2O2.

3. The reaction 2NO(g) + Cl2(g) 2NOCl(g) is second order in NO and first order in Cl2. In a volume of2 dm3, 5 mole of nitric oxide and 2 mol of Cl2 were brought together, and the initial rate was 2.4 × 10 3 moledm3 s. What will be the rate when half of the chlorine has reacted ?

ETOOSINDIA.COMIndia's No.1 Online Coaching for JEE Main & Advanced

3rd Floor, H.No.50 Rajeev Gandhi Nagar, Kota, Rajasthan 324005HelpDesk : Tel. 092142 33303

Page 32: Contentsfile.etoosindia.com/sites/default/files/study...Topic Page No. Theory 01 - 06 Exercise - 1 07 - 24 Exercise - 2 25 - 33 Exercise - 3 34 - 38 Exercise - 4 39 - 41 Answer Key

CHEMICAL KINETICS # 31

4. For the reaction 3BrO(aq) BrO3 (aq) + 2Br (aq) in alkaline solution, the value of rate constant at

800C in the rate law for [BrO]/t was found to be 0.057 L.mol1.s1. What is the rate constant when the ratelaw is written for (a) [BrO3

]/t (b) + [Br ] / t ?

5. The rate law of a chemical reaction given below :

2NO (g) + O2 (g) 2NO2 (g)

is given as rate = k [NO]2 [O2]. How will the rate of reaction change if the volume of reaction vessel is reduced to1/4th of its original value?

6. Decomposition of ammonia on platinum surface follow the change,

2NH3 (g) N2 (g) + 3H2 (g)

(a) What does dt

]NH[d 3 denote ?

(b) What does dt

]N[d 2 and dt

]H[d 2 denote?

(c) If the decomposition is zero order then what are the rate of production of N2 and H2 if k = 2.5 × 10�4 Ms�1 ?

(d) If the rate obeys �dt

]NH[d 3 = ]NH[k1]NH[k

32

31

, what will be the order for decomposition of NH3 if (i) [NH3] is very

very less and (ii) [NH3] is very very high K1 and K2 are constant.

7. Substance A reacts according to a first order rate law with k = 5 x 105 s1.

(a) If the initial concentration of A is 1.0 M, what is initial rate .

(b) What is the rate after 1.0 hour. (Given e�0.18 = 0.84)

8. Consider the following data on the hypothetical reaction between the reactants (A) and (B) give products2A + 2B C + 3D [A] [B] Rate r[mol �1] [mol �1] [mol �1 s�1]

1. 6.0 x 10-3 1.0 x 10-3 0.0122. 6.0 x 10-3 2.0 x 10-3 0.0243. 2.0 x 10-3 1.4 x 10-3 0.00204. 4.0 x 10-3 1.4 x 10-3 0.0080If the reaction rate given by r = [A]a[B]b, then,(i) What is the order of the reaction with respect to A. (ii) What is the order of the reaction with respect to B.(iii) What is the overall order. (iv) Write the rate law equation.(v) Calculate the rate constant.

9. The reaction A + 2B C + 2D is run three times. In the second run, the initial concentration of A is double thatin the first run, and the initial rate of the reaction is double that of the first run. In the third run, the initialconcentration of each reactant is double the respective concentrations in the first run, and the initial rate isdouble that of the first run. What is the order of the reaction with respect to each reactant?

10. In a kinetic study of the reduction of nitric oxide with hydrogen, the initial pressure of 340 mm, an equimolarmixture of gases was reduced to half the value in 102 seconds. In another experiment, the initial pressure of 288mm, under the same conditions was reduced to half the value in 140 sec. Calculate the order of the reaction.

11. (a) The half life period and initial concentration for a reaction are as follows. What is order of reaction?Initial concentration 350 540 158t1/2 425 275 941

(b) The half-life period for the thermal decomposition of PH3 at three different pressures are given belowInitial pressure (mm Hg) 707 79 37.5Half--life (seconds) 84 84 83What is the order of the reaction ?

ETOOSINDIA.COMIndia's No.1 Online Coaching for JEE Main & Advanced

3rd Floor, H.No.50 Rajeev Gandhi Nagar, Kota, Rajasthan 324005HelpDesk : Tel. 092142 33303

Page 33: Contentsfile.etoosindia.com/sites/default/files/study...Topic Page No. Theory 01 - 06 Exercise - 1 07 - 24 Exercise - 2 25 - 33 Exercise - 3 34 - 38 Exercise - 4 39 - 41 Answer Key

CHEMICAL KINETICS # 32

12. The decomposition of Cl2O7 at 400 K in gaseous phase to Cl2 and O2 is of order reaction. After 55 sec at 400

K, the pressure of reaction mixture increases from 0.62 to 1.88 atm. Calculate the rate constant of given

reaction, also calculate the pressure of reaction mixture after 100 second.

[Given : n 1331

= 0.869, e1.58 = 4.858] CI2O

7(g) CI

2 (g) +

27

O2 (g)

13. For a homogeneous gaseous phase reaction 2A 3B + C the initial pressure was P0 while pressure after time

't' was P. Find the pressure after time '2t'. Assume first order reaction.

14. The kinetic of hydrolysis of methyl acetate in excess dilute HCl at 25°C were followed by withdrawing 2 mL of the

reaction mixture at intervals of (t), adding 50 mL water and titrating with baryta water. Determine the velocity

constant of hydrolysis.

t (in minute) 0 75 119 259

Titre value (in mL) 19.24 24.20 26.60 32.23 42.03

[ln 83.1779.22

= 0.2454, ln 43.1579.22

= 0.39, ln 8.979.22

= 0.8439]

15. A D B + C

Time 0 t

Volume of reagent V1 V2 V3

The reagent reacts with only B, C and D. Find k.

16. A first order reaction, A B, requires activation energy of 70 kJ mol1. When a 20% solution of A was kept at

250C for 20 minutes, 25% decomposition took place. What will be the percent decomposition in the same time

in a 30% solution maintained at 40 0C? Assume that activation energy remains constant in this range of

temperature.

17. The thermal decomposition of N2O5 occurs in the following steps.

Step - N2O5 slow NO2 + NO3

Step - N2O5 + NO3 fast 3NO2 + O2

2N2O5 4NO2 + O2

suggest the rate expression.

18. The reaction 2NO (g) + Br2 (g) 2NOBr (g), obeys the following mechanism.

(i) NO (g) + Br2 (g) NOBr2 (g)

(ii) NOBr2 (g) + NO (g) Slow 2NOBr (g)

Suggest the rate expression.

ETOOSINDIA.COMIndia's No.1 Online Coaching for JEE Main & Advanced

3rd Floor, H.No.50 Rajeev Gandhi Nagar, Kota, Rajasthan 324005HelpDesk : Tel. 092142 33303

Page 34: Contentsfile.etoosindia.com/sites/default/files/study...Topic Page No. Theory 01 - 06 Exercise - 1 07 - 24 Exercise - 2 25 - 33 Exercise - 3 34 - 38 Exercise - 4 39 - 41 Answer Key

CHEMICAL KINETICS # 33

19. Given the following steps in the mechanism for a chemical reaction :

A + B C (fast)

B + C D + E slow)

D + F A + E fast)

At any time [C] is directly proportional to [A].

(a) What is the stoichiometric equation for the reaction ?

(b) Which species, if any, are catalysts in this reaction ?

(c) Which species, if any, are intermediates in this reaction?

(d) Write the rate law for the rate-determining step.

(e) Write the rate law for this reaction.

(f) What is overall order of the reaction ?

20. The forward reactions rate for the nitric oxide-oxygen reaction

2NO + O2 2NO2 has the rate law ; rate = K [NO]2 [O2] .

Assume the mechanism:

2NO N2O2 , (rapid equilibration) ;

N2O2 + O2 'k 2 NO2 [slow step].

Show that the rate law can be explained, where in K = kk.

21. Experiment shows that the equilibrium constant of the reaction :

C2H5OH + CH3COOH CH3COOC2H5 + H2O

is 2.8 at room temperature, and the velocity constant of the forward reaction is 0.002. When a catalyst is added

this velocity constant is increased to 0.0045. What is now the velocity constant of the backward reaction?

22. For a first order reversible reaction A B , the initial concentration of A and B are [A]0 and zero

respectively. If concentrations at equilibrium are [A]eq. and [B]eq. derive an expression for the time taken by B to

attain concentration equal to [B]eq/2 .

ETOOSINDIA.COMIndia's No.1 Online Coaching for JEE Main & Advanced

3rd Floor, H.No.50 Rajeev Gandhi Nagar, Kota, Rajasthan 324005HelpDesk : Tel. 092142 33303

Page 35: Contentsfile.etoosindia.com/sites/default/files/study...Topic Page No. Theory 01 - 06 Exercise - 1 07 - 24 Exercise - 2 25 - 33 Exercise - 3 34 - 38 Exercise - 4 39 - 41 Answer Key

CHEMICAL KINETICS # 34

PART - I : IIT-JEE PROBLEMS (PREVIOUS YEARS)

* Marked Questions are having more than one correct option.

1. The rate constant for the reaction,2N2O

5 4NO

2+ O

2, is 3 ×10�5 s�1. If the rate is 2.4 × 10�5 mol L�1 s�1, then

the concentration of N2O

5 (in mol L�1) is : [JEE-2000, 1/35]

(A) 1.4 (B) 1.2 (C) 0.04 (D) 0.8

2. A hydrogenation reaction is carried out at 500 K. If the same reaction is carried out in the presence of a catalystat the same rate, the temperature required is 400 K. Calculate the activation energy of the reaction if the catalystlowers the activation barrier by 20 kJ mol�1. [JEE-2000, 3/100]

3. If '' is the intensity of absorbed light and 'C' is the concentration of AB for the photochemical process,

AB + hv *AB , the rate of formation of *AB is directly proportional to [JEE-2001, 1/35](A) C (B) (C) 2 (D) C.

4. The rate of a first order reaction is 0.04 mole litre�1 s�1 at 10 minutes and 0.03 mol litre�1 s�1 at 20 minutes afterinitiation. Find the half life of the reaction. [JEE-2001, 5/100]

5. Consider the chemical reaction,N2(g) + 3H2(g) 2NH3(g)

The rate of this reaction can be expressed in terms of time derivatives of conc. of N2(g) , H2(g) or NH3(g). Identifythe correct relationship amongst rate expressions : [JEE-2002, 3/90]

(A) Rate = dt

]NH[d21

dt]H[d

31

dt]N[d 322 (B) Rate =

dt]NH[d

2dt

]H[d3

dt]N[d 322

(C) Rate = dt

]NH[d21

dt]H[d

31

dt]N[d 322 (D) Rate =

dt]NH[d

dt]H[d

dt]N[d 322

6. In the biologically-catalysed oxidation of ethanol, the concentration of ethanol decreases in a first order reactionfrom 800 mol dm�3 to 50 mol dm�3 in 2 ×104 s. The rate constant (s�1) of the reaction is : [JEE-2003, 3/84](A) 3.45 × 10�5 (B) 1.38 × 10�4 (C) 1.00 × 10�4 (D) 5.00 × 10�5

7. Given X product (Taking 1st order reaction)

conc 0.01 0.0025(mol/lit)time (min) 0 40Initial rate of reaction is in mol //min. [JEE-2004, 3/84](A) 3.43 × 10�4 (B) 1.73 × 10�4 (C) 3.43 × 10�5 (D) 1.73 × 10�5

8. Initial rates, r0, of the A + B P at different initial concentrations of A and B ([A]0 and [B]0) are given below:

��������������������������������������������

[A]0 [B]0 r0

(mol L�1) (mol L�1) (mol L�1 s�1)��������������������������������������������

0.1 0.1 0.050.2 0.1 0.100.1 0.2 0.05

��������������������������������������������

(a) Write the rate equation.(b) Calculate the rate constant of the reaction. [JEE-2004, 2/60]

ETOOSINDIA.COMIndia's No.1 Online Coaching for JEE Main & Advanced

3rd Floor, H.No.50 Rajeev Gandhi Nagar, Kota, Rajasthan 324005HelpDesk : Tel. 092142 33303

Page 36: Contentsfile.etoosindia.com/sites/default/files/study...Topic Page No. Theory 01 - 06 Exercise - 1 07 - 24 Exercise - 2 25 - 33 Exercise - 3 34 - 38 Exercise - 4 39 - 41 Answer Key

CHEMICAL KINETICS # 35

9. For a reaction 2X(g) 3Y(g) + 2Z(g) the following data is obtained.

Time (min)Px (mm of Hg)

(Partial pressure of X)0 800

100 400200 200

Find order with respect to X, rate constant of the reaction, time taken for 75% completion and find the totalpressure when partial pressure of X, Px = 700 mm of Hg. [JEE-2005, 2/60]

10. Consider a reaction aG + bH Products. When concentration of both the reactants G and H is doubled, therate increases by eight times. However, when concentration of G is doubled keeping the concentration of Hfixed, the rate is doubled. The overall order of the reaction is : [JEE-2007, 3/81](A) 0 (B) 1 (C) 2 (D) 3

11. Under the same reaction conditions, initial concentration of 1.386 mol dm�3 of a substance becomes half in 40

seconds and 20 seconds through first order and zero order kinetics, respectively. Ratio

0

1

k

k of the rate

constant for first order (k1) and zero order (k

0) of the reaction is. [JEE-2008, 3/82]

(A) 0.5 mol�1 dm3 (B) 1.0 mol dm�3 (C) 1.5 mol dm�3 (D) 2.0 mol�1 dm3

12. For a first order reaction A P, the temperature (T) dependent rate constant (k) was found to follow the equation

log k = � (2000) T1

+ 6.0. The pre-exponential factor A and the activation energy Ea, respectively, are :

[JEE-2009, 3/80](A) 1.0 × 106 s�1 and 9.2 kJ mol�1 (B) 6.0 s�1 and 16.6 kJ mol�1

(C) 1.0 × 106 s�1 and 16.6 kJ mol�1 (D) 1.0 × 106 s�1 and 38.3 kJ mol�1

13. Plots showing the variation of the rate constant (k) with temperature (T) are given below. The plot that follows

Arrhenius equation is : [JEE-2010, 3/163]

(A) (B) (C) (D)

14. The concentration of R in the reaction R P was measured as a function of time and the following data isobtained : [JEE-2010, 3/163]

18.012.005.00.0.)(mint

10.040.075.00.1)molar](R[

The order of the reaction is :

15. The number of neutrons emitted when U23592 undergoes controlled nuclear fission to Xe142

54 and Sr9038 is :

[JEE-2010, 3/163]

ETOOSINDIA.COMIndia's No.1 Online Coaching for JEE Main & Advanced

3rd Floor, H.No.50 Rajeev Gandhi Nagar, Kota, Rajasthan 324005HelpDesk : Tel. 092142 33303

Page 37: Contentsfile.etoosindia.com/sites/default/files/study...Topic Page No. Theory 01 - 06 Exercise - 1 07 - 24 Exercise - 2 25 - 33 Exercise - 3 34 - 38 Exercise - 4 39 - 41 Answer Key

CHEMICAL KINETICS # 36

16. Bombardment of aluminum by -particle leads to its artificial disintegration into two ways, (i) and (ii) as shown.Products X, Y and Z respectivey are : [JEE-2011, 3/163]

(A) Proton, neutron, position (B) Neutron, position, proton(C) proton, positron, neutron (D) positron, proton, neutorn

17. For the first order reaction [JEE-2011, 4/163]2N

2O

5(g) 4NO

2(g) + O

2(g)

(A) the concentration of the reactant decreases exponentially with time(B) the half-life of the reaction decreases with increasing temperature(C) the half-life of the reaction depends on the initial concentration of the reactant(D) the reaction proceeds to 99.6% completion in eight half-life duration

18. An organic compound undergoes first-order decomposition. The time taken for its decompositionto 1/8 and 1/10

of its initial concentration are t1/8 and t1/10 respectively. What is the value of]t[

]t[

10/1

8/1 × 10? (take log10 2 = 0.3).

[IIT-JEE-2012]

PART - II : AIEEE PROBLEMS (PREVIOUS YEARS)

1. Units of rate constant of first and zero order reactions in terms of molarity M unit are respectively [AIEEE- 2002]

(1) sec�1, M sec�1 (2) sec�1, M (3) M.sec�1, sec�1 (4) M, sec�1

2. For the reaction A + 2B C, rate is given by R = [A] [B]2 then the order of the reaction is : [AIEEE- 2002](1) 3 (2) 6 (3) 5 (4) 7

3. The differential rate law for the reaction H2 + I

2 2HI is : [AIEEE- 2002]

(1)

dt]HI[d

dt

]I[d

dt

Hd 22 (2)

dt]HI[d

21

dt]I[d

dtHd 22

(3)

dt]HI[d

dt]I[d

21

dtHd

21 22 (4)

dt

]HI[ddt

]I[d2

dtHd

2 22

4. The rate law for a reaction between the substances A and B is given by rate = k [A]n [B]m. On doubling theconcentration of A and halving the concentration of B, the ratio of the new rate to the earlier rate of the reactionwill be as [AIEEE- 2003]

(1)nm2

1

(2) (m + n) (3) (n � m) (4) 2(n � m).

5. For the reaction system: 2NO(g)

+ O2(g)

2NO2(g)

, volume is suddenly reduced to half its value by increasing

the pressure on it. If the reaction is of first order with respect to O2 and second order with respect to NO, the rate

of reaction will : [AIEEE- 2003](1) diminish to one-fourth of its initial value (2) diminish to one-eighth of its initial value(3) increase to eight times of its initial value (4) increase to four times of its initial value.

ETOOSINDIA.COMIndia's No.1 Online Coaching for JEE Main & Advanced

3rd Floor, H.No.50 Rajeev Gandhi Nagar, Kota, Rajasthan 324005HelpDesk : Tel. 092142 33303

Page 38: Contentsfile.etoosindia.com/sites/default/files/study...Topic Page No. Theory 01 - 06 Exercise - 1 07 - 24 Exercise - 2 25 - 33 Exercise - 3 34 - 38 Exercise - 4 39 - 41 Answer Key

CHEMICAL KINETICS # 37

6. In the respect of the equation k = Ae �Ea/RT in chemical kinetics, which one of the following statements is correct :

[AIEEE- 2003]

(1) k is equilibrium constant (2) A is adsorption factor

(3) Ea is energy of activation (4) R is Rydberg constant.

7. In a first order reaction, the concentration of the reactant, decreases from 0.8 M to 0.4 M in 15 minutes. The time

taken for the concentration to change from 0.1 M to 0.025 M is : [AIEEE- 2004]

(1) 30 minutes (2) 15 minutes (3) 7.5 minutes (4) 60 minutes

8. The rate equation for the reaction 2A + B C is found to be : rate = k[A] [B]. The correct statement in

relation to this reaction is that the : [AIEEE- 2004]

(1) unit of k must be sec�1

(2) t1/2

is a constant

(3) rate of formation of C is twice the rate of disappearance of A

(4) value of k is independent of initial concentrations of A and B.

9. The half - life of a radioisotope is four hours. If the initial mass of the isotope was 200 g, the mass remaining after

24 hours undecayed is : [AIEEE- 2004]

(1) 1.042 g (2) 2.084 g (3) 3.125 g (4) 4.167 g.

10. Consider an endothermic reaction X Y with the activation energies Eb and E

f for the backward and forward

reaction, respectively. In general [AIEEE- 2005]

(1) Eb < E

f(2) H = U (3) H < U (4) H > U

11. A reaction involving two different reactants can never be : [AIEEE- 2005]

(1) unimolecular reaction (2) first order reaction

(3) second order reaction (4) bimolecular reaction

12. A reaction was found to be second order with respect to the concentration of carbon monoxide. If the concentration

of carbon monoxide is doubled, with everything else kept the same, the rate of reaction will be : [AIEEE- 2006]

(1) remain unchanged (2) tripled

(3) increased by a factor of 4 (4) doubled

13. The following mechanism has been proposed for the reaction of NO with Br2 to form NOBr.

NO (g) + Br2 (g) NOBr

2 (g)

; NOBr

2 (g) + NO (g) 2NOBr (g) (slow step)

If the second step is the rate determining step, the order of the reaction with respect to NO(g) is

[AIEEE- 2007, 3/120]

(1) 1 (2) 0 (3) 3 (4) 2

14. The energies of activation for forward and reverse reactions for A2 + B

2 2AB are 180 kJ mol�1 and

200 kJ mol�1 respectively. The presence of a catalyst lowers the activation energy of both (forward and reverse)

reactions by 100 kJ mol�1. The enthalpy change of the reaction (A2 + B

2 2AB) in the presence of catalyst will be

(in kJ mol�1). [AIEEE- 2007, 3/120]

(1) 280 (2) 20 (3) 300 (4) 120

ETOOSINDIA.COMIndia's No.1 Online Coaching for JEE Main & Advanced

3rd Floor, H.No.50 Rajeev Gandhi Nagar, Kota, Rajasthan 324005HelpDesk : Tel. 092142 33303

Page 39: Contentsfile.etoosindia.com/sites/default/files/study...Topic Page No. Theory 01 - 06 Exercise - 1 07 - 24 Exercise - 2 25 - 33 Exercise - 3 34 - 38 Exercise - 4 39 - 41 Answer Key

CHEMICAL KINETICS # 38

15. A radioactive element gets spilled over the floor of a room. Its half-life period is 30days. If the initial activity is ten

times the permissible value, after how many days will it be safe to enter the room :

[AIEEE- 2007, 3/120]

(1) 10 days (2) 100 days (3) 1000 days (4) 300 days

16. For a reaction21

A 2B, rate of disappearance of 'A' related to the rate of appearance of 'B' by the

expression. [AIEEE- 2008, 3/105]

(1) dt

]B[d41

dt]A[d (2)

dt]B[d

dt]A[d (3)

dt]B[d

4dt

]A[d (4)

dt]B[d

21

dt]A[d

17. The half life period of a first order chemical reaction is 6.93 minutes. Time required for the completion of 99% of

the chemical reaction will be (log 2 = 0.301) : [AIEEE - 2009, 8/144]

(1) 23.03 minutes (2) 46.06 minutes (3) 460.6 minutes (4) 230.3 minutes

18. The time for half life period of a certain reaction A Products is 1 hour. When the initial concentration of the

reactant �A�, is 2.0 mol L�1 , how much time does it take for its concentration to come from 0.50 to 0.25 mol L�1. If

it is a zero order reaction? [AIEEE - 2010, 8/144]

(1) 4 h (2) 0.5 h (3) 0.25 h (4) 1 h

19. Consider the reaction [AIEEE - 2010, 4/144]

Cl2 (aq) + H

2S(aq) S(s) + 2H+ (aq) + 2Cl� (aq)

The rate equation for this reaction is

rate = k [Cl2][H

2S]

Which of these mechanisms is/are consistent with this rate equation?

A. Cl2 + H

2S H+ + Cl� + Cl+ + HS� (slow)

Cl+ + HS� H+ + Cl� + S (fast)

B. H2S H+ + HS� (fast equilibrium)

Cl2 + HS� 2Cl� + H+ + S (slow)

(1) B only (2) Both A and B (3) Neither A nor B (4) A only

20. The rate of a chemical reaction doubles for every 10° C rise of temperature. If the temperature is raised by 50° C,

the rate of the reaction increases by about : [AIEEE-2011]

(1) 10 times (2) 24 times (3) 32 times (4) 64 times

21. For a first order reaction, (A) products, the concentration of A changes from 0.1 M to 0.025 M in 40

minutes. The rate of reaction when the concentration of A is 0.01 M, is [AIEEE-2012, 4/120]

(1) 1.73 × 10�3 M/min (2) 3.47 × 10�4 M/min (3) 3.47 × 10�5 M/min (4) 1.73 × 10�4 M/min

ETOOSINDIA.COMIndia's No.1 Online Coaching for JEE Main & Advanced

3rd Floor, H.No.50 Rajeev Gandhi Nagar, Kota, Rajasthan 324005HelpDesk : Tel. 092142 33303

Page 40: Contentsfile.etoosindia.com/sites/default/files/study...Topic Page No. Theory 01 - 06 Exercise - 1 07 - 24 Exercise - 2 25 - 33 Exercise - 3 34 - 38 Exercise - 4 39 - 41 Answer Key

PART-NCERT QUESTIONS

1. From the rate expression for the following reactions, determine their order of reaction and the dimensions of therate constants.(i) 3NO(g) N

2O (g) Rate = k[NO]2

(ii) H2O

2 (aq) + 3I� (aq) + 2H+ 2H

2O (l) + I

3� Rate = k[H

2O

2] [I�]

(iii) CH3CHO (g) CH

4 (g) + CO(g) Rate = k[CH

3CHO]3/2

(iv) C2H

5Cl (g) C

2H

4 (g) + HCl (g) Rate = k[C

2H

5Cl]

2. For the reaction :2A + B A2B

the rate = k[A][B]2 with k = 2.0 × 10�6 mol�2 L2 s�1. Calculate the initial rate of the reaction when[A] = 0.1 mol L�1, [B] = 0.2 mol L�1. Calculate the rate of reaction after [A] is reduced to 0.06 mol L�1.

3. The decomposition of NH3 on platinum surface is zero order reaction. What are the rates of production of N

2 and

H2 if k = 2.5 × 10�4 mol�1 L s�1?

4. The decomposition of dimethyl ether leads to the formation of CH4, H

2 and CO and the reaction rate is given by

Rate = k [CH3OCH

3]3/2

The rate of reaction is followed by increase in pressure in a closed vessel, so the rate can also be expressed interms of the partial pressure of dimethyl ether, i.e.,

Rate = k (PCH3OCH3

)3/2

If the pressure is measured in bar and time in minutes, then what are the units of rate and rate constants?

5. Mention the factors that affect the rate of a chemical reaction.

6. A reaction is second order with respect to a reactant. How is the rate of reaction affected if the concentration ofthe reactant is(i) doubled (ii) reduced to half ?

7. What is the effect of temperature on the rate constant of a reaction? How can this effect of temperature on rateconstant be represented quantitatively?

8. In a pseudo first order hydrolysis of ester in water, the following results were obtained :

(i) Calculate the average rate of reaction between the time interval 30 to 60 seconds.(ii) Calculate the pseudo first order rate constant for the hydrolysis of ester.

9. A reaction is first order in A and second order in B.(i) Write the differential rate equation.(ii) How is the rate affected on increasing the concentration of B three times?(iii) How is the rate affected when the concentrations of both A and B are doubled?

10. In a reaction between A and B, the initial rate of reaction (r0) was measured for different initial concentrations of

A and B as given below :

What is the order of the reaction with respect to A and B?

CHEMICAL KINETICS # 39

ETOOSINDIA.COMIndia's No.1 Online Coaching for JEE Main & Advanced

3rd Floor, H.No.50 Rajeev Gandhi Nagar, Kota, Rajasthan 324005HelpDesk : Tel. 092142 33303

Page 41: Contentsfile.etoosindia.com/sites/default/files/study...Topic Page No. Theory 01 - 06 Exercise - 1 07 - 24 Exercise - 2 25 - 33 Exercise - 3 34 - 38 Exercise - 4 39 - 41 Answer Key

11. The following results have been obtained during the kinetic studies of the reaction :2A + B C + D

Determine the rate law and the rate constant for the reaction.

12. The reaction between A and B is first order with respect to A and zero order with respect to B. Fill in the blanksin the following table:

13. Calculate the half-life of a first order reaction from their rate constants given below :(i) 200 s�1 (ii) 2 min�1 (iii) 4 years�1

14. The half-life for radioactive decay of 14C is 5730 years. An archaeological artifact containing wood had only 80%of the 14C found in a living tree. Estimate the age of the sample.

15. The experimental data for decomposition of N2O

5

[2N2O

5 4NO

2 + O

2] in gas phase at 318K are given below :

(i) Plot [N2O

5] against t.

(ii) Find the half-life period for the reaction.(iii) Draw a graph between log[N

2O

5] and t.

(iv) What is the rate law ?(v) Calculate the rate constant.(vi) Calculate the half-life period from k and compare it with (ii).

16. The rate constant for a first order reaction is 60 s�1. How much time will it take to reduce the initial concentrationof the reactant to its 1/16th value?

17. During nuclear explosion, one of the products is 90Sr with half-life of 28.1 years. If 1µg of 90Sr was absorbed in thebones of a newly born baby instead of calcium, how much of it will remain after 10 years and 60 years if it is notlost metabolically.

CHEMICAL KINETICS # 40

ETOOSINDIA.COMIndia's No.1 Online Coaching for JEE Main & Advanced

3rd Floor, H.No.50 Rajeev Gandhi Nagar, Kota, Rajasthan 324005HelpDesk : Tel. 092142 33303

Page 42: Contentsfile.etoosindia.com/sites/default/files/study...Topic Page No. Theory 01 - 06 Exercise - 1 07 - 24 Exercise - 2 25 - 33 Exercise - 3 34 - 38 Exercise - 4 39 - 41 Answer Key

18. For a first order reaction, show that time required for 99% completion is twice the time required for the completionof 90% of reaction.

19. A first order reaction takes 40 min for 30% decomposition. Calculate t1/2

.

20. For the decomposition of azoisopropane to hexane and nitrogen at 543 K, the following data are obtained.Calculate the rate constant.

21. The following data were obtained during the first order thermal decomposition of SO2Cl

2 at a constant volume.

SO2 Cl

2 (g) SO

2 (g) + Cl

2 (g)

Calculate the rate of the reaction when total pressure is 0.65 atm.

22. The rate constant for the decomposition of N2O

5 at various temperatures is given below :

Draw a graph between ln k and 1/T and calculate the values of A and Ea. Predict the rate constant at 30° and

50°C.

23. The rate constant for the decomposition of hydrocarbons is 2.418 × 10�5 s�1 at 546 K. If the energy of activationis 179.9 kJ/mol, what will be the value of pre-exponential factor.

24. Consider a certain reaction A Products with k = 2.0 × 10�2 s�1. Calculate the concentration of A remaining after100 s if the initial concentration of A is 1.0 mol L�1.

25. Sucrose decomposes in acid solution into glucose and fructose according to the first order rate law, witht1/2

= 3.00 hours. What fraction of sample of sucrose remains after 8 hours ?

26. The decomposition of hydrocarbon follows the equation k = (4.5 × 1011s�1) e�28000K/T. Calculate Ea.

k = (4.5 × 1011s�1) e�28000K/T

27. The rate constant for the first order decomposition of H2O

2 is given by the following equation :

log k = 14.34 � 1.25 × 104K/TCalculate E

a for this reaction and at what temperature will its half-period be 256 minutes?

28. The decomposition of A into product has value of k as 4.5 × 103 s�1 at 10°C and energy of activation

60 kJ mol�1. At what temperature would k be 1.5 × 104 s�1?

29. The time required for 10% completion of a first order reaction at 298K is equal to that required for its 25%completion at 308K. If the value of A is 4 × 1010 s�1. Calculate k at 318 K and E

a.

30. The rate of a reaction quadruples when the temperature changes from 293 K to 313 K. Calculate the energy ofactivation of the reaction assuming that it does not change with temperature.

CHEMICAL KINETICS # 41

ETOOSINDIA.COMIndia's No.1 Online Coaching for JEE Main & Advanced

3rd Floor, H.No.50 Rajeev Gandhi Nagar, Kota, Rajasthan 324005HelpDesk : Tel. 092142 33303

Page 43: Contentsfile.etoosindia.com/sites/default/files/study...Topic Page No. Theory 01 - 06 Exercise - 1 07 - 24 Exercise - 2 25 - 33 Exercise - 3 34 - 38 Exercise - 4 39 - 41 Answer Key

CHEMICAL KINETICS # 42

EXERCISE # 1PART # I

A-1. (B) A-2. (C) A-3. (B) A-4. (A) A-5. (D) A-6. (D) A-7. (B)A-8. (B) A-9. (D) A-10. (C) B-1. (B) B-2. (D) B-3. (D) B-4. (C)B-5. (D) B-6. (B) B-7. (B) B-8. (B) B-9. (B) B-10. (D) B-11. (D)B-12. (A) C-1. (A) C-2. (D) C-3. (B) C-4. (D) C-5. (B) C-6. (B)C-7. (D) C-8. (B) C-9. (D) C-10. (C) C-11. (C) C-12. (C) C-13. (B)C-14. (C) C-15. (B) C-16. (B) C-17. (B) C-18. (B) C-19. (A) C-20. (B)C-21. (C) C-22. (C) C-23. (C) C-24. (C) C-25. (C) C-26. (A) C-27. (C)D-1. (B) D-2. (B) D-3. (D) D-4. (D) D-5. (A) D-6. (C) D-7. (A)D-8. (D) D-9. (D) D-10. (C) D-11. (C) D-12. (C) D-13. (B) E-1. (A)E-2. (B) E-3. (B) E-4. (A) E-5. (B) E-6. (B) F-1. (A) F-2. (C)F-3. (A) F-4. (A) F-5. (C) F-6. (C) F-7. (D) F-8. (A) F-9. (A)F-10. (B) F-11. (A) F-12. (D) F-13. (C) F-14. (B) F-15. (D) G-1. (C)G-2. (D) G-3. (B) G-4. (D) G-5. (C) G-6. (A) G-7.* (ABCD) H-1. (B)H-2. (B) H-3. (B) H-4. (B) H-5. (B) H-6. (B)

PART # II1. (A*) 2. (D*) 3. (B*) 4. (A*) 5. (C*) 6. (A*) 7. (C*)8. (C*) 9. (C*) 10. (B*) 11. (B*) 12. (A*) 13. (C*) 14. (C*)15. (A) r ; (B) p ; (C) s; (D) q 16. (A) p, q, r, s ; (B) q, r, s ; (C) p, q, r, s ; (D) p, r, s17. (A) s, (B) r, (C) p, (D) q 18. (A) r ; (B) s ; (C) p ; (D) q19. (C) 20. (C) 21. (D) 22. (A) 23. (C) 24. (A) 25. (C)26. (A) 27. (B) 28. (A) 29. (A) 30. (B) 31. F 32. T33. F 34. T 35. F 36. F 37. T 38. T 39. F40. T 41. T 42. T 43. F 44. F 45. F 46. T47. T 48. T 49. F 50. T 51. T 52. T 53. F54. T 55. T 56. F 57. F 58. T 59. T 60. F61. threshold energy 62. half 63. equal to H 64. directly proportional65. mol L�1s�1 66. rare 67. lowering 68. fast 69. slowest70. increases 71. 3 72. rate constant 73. 2 and 3

74. rate = k[M]2 75.2

11 76. � k 77. Activation energy for forward reaction

78. infinte temp. or for zero Ea 79. first 80.303.2

k 81. unchanged

82. sum, exponents

EXERCISE # 21. (C) 2. (C) 3. (C) 4. (D) 5. (B) 6. (B) 7. (A)8. (D) 9. (C) 10. (B) 11. (B) 12. (C) 13. (D) 14. (A)15. (B) 16. (B) 17. (D) 18. (D) 19. (B) 20. (B) 21. (D)22. (C) 23. (C) 24. (D) 25. (C) 26. (A) 27. (A) 28. (B)29. (D) 30. (C) 31. (C) 32. (A) 33. (B) 34. (C) 35. (D)36. (D) 37. (C) 38. (C) 39. (D) 40. (AD) 41. (ACD) 42. (AC)

43. (ABC) 44. (CD) 45. (ABC)

SUBJECTIVE1. (a) 4.5 × 10�3 M sec�1 (b) 3.0 × 10�3 M sec�1 2. (a) 40.5 gm min�1 (b) 76.5 gm min�1

3. 4.32 × 10�4 M sec�1 4. (a) 0.019 M1 s1 , (b) 0.038 M1 s1

5. 646. (a) Rate of decomposition of NH3. (b) Rate of formation of N2 and rate of formation of H2

(c) 2.5 × 10�4 M sec�1 , 7.5 × 10�4 M sec�1 (d) (i) 1 , (ii) 07. (a) 5 × 105 M/s (b) 4.2 × 105 M/s

ETOOSINDIA.COMIndia's No.1 Online Coaching for JEE Main & Advanced

3rd Floor, H.No.50 Rajeev Gandhi Nagar, Kota, Rajasthan 324005HelpDesk : Tel. 092142 33303

Page 44: Contentsfile.etoosindia.com/sites/default/files/study...Topic Page No. Theory 01 - 06 Exercise - 1 07 - 24 Exercise - 2 25 - 33 Exercise - 3 34 - 38 Exercise - 4 39 - 41 Answer Key

CHEMICAL KINETICS # 43

8. (i) 2nd order, (ii) 1st order, (iii) Third order, (iv) r = K[A]2[B] (v) k = 3.33 × 105 mole�2 +2 sec�1

9. order with respect to A = 1, order with respect to B = 0

10. 3 11. (a) n = 2, (b) First Order

12. 1.58 × 10�2, 2.33 atm. 13. 2P0 0

20

P

)PP2( 14. 3.27 × 10�3 min�1

15.t1

ln )VV()VV(

23

13

16. % decomposition = 67.10% 17. k [N2O5]

18. Rate = k [NO]2 [Br2] 19. (a) 2 B+F 2E; (b) A; (c) C,D; (d) rate = k [B][C]; (e) rate = k'[A][B] (f) 2.

21. 0.001607 22. t =bf KK

693.0

EXERCISE # 3

PART-I1.1 (D) 1.2 EA = 100 KJ/mol 1.3 (B) 1.4 t1/2 = 24 min. 1.5 (A) 1.6 (B)

1.7 (A) 1.8 (a) R0 = k[A0], (b) 0.5 sec�1.

1.9 2X(g) 3Y(g) + 2Z(g)t = 0 800 � �

t = 0 800 � 2x 3x 2x = (800 + 3x)from given data in time 100 min the partial pressure ofX decreases from 800 to 400so t1/2 100 min. Also in next 100 min Px decreasesfrom 400 to 200 to again t1/2 = 100 min. Since half leftis independent of initial concentration so reaction mustIst order with respect to X.

Rate constant K = 2/1t2n

= 6.93 × 10�3 min�1.

Time taken for 75% completion = 2 × t1/2 = 200 min.Now when Px = 700 = 800 � 2x so x = 50 mm of Hg

so total pressure = 800 + 3x = 950 mm of Hg1.10 (D) 1.11 (A) 1.12 (D) 1.13 (A) 1.14 0 1.15 31.16 (A) 1.17 (A, B, D) 1.18 9

PART-II

2.1 (1) 2.2 (1) 2.3 (4) 2.4 (4) 2.5 (3) 2.6 (3) 2.7 (1)2.8 (4) 2.9 (3) 2.10 (1) 2.11 (1) 2.12 (3) 2.13 (4) 2.14 (2)2.15 (2) 2.16 (1) 2.17 (2) 2.18 (3) 2.19 (4) 2.20 (3) 2.21 (2)

EXERCISE # 4

1. rav

= 6.66 × 10�6 Ms�1 2. Rate of reaction = rate of diappearance of A = 0.005 mol litre�1min�1

3. Order of the reaction is 2.54. X Y Rate = k[X]2

The rate will increase 9 times5. t = 444 s 6. 1.925 × 10�4 s�1 8. Ea = 52.897 kJ mol�1

9. 1.471 × 10�19

ETOOSINDIA.COMIndia's No.1 Online Coaching for JEE Main & Advanced

3rd Floor, H.No.50 Rajeev Gandhi Nagar, Kota, Rajasthan 324005HelpDesk : Tel. 092142 33303